Крок 1 - Медицина 2017 (буклет)

1 / 200
Тривале перебування в умовах спеки викликало у людини спрагу. Сигналізація від яких рецепторів, перш за все, зумовила її розвиток? Prolonged stay in hot conditions made a person thirsty. Signaling from which receptors primarily determined its development?

Натрiєвi рецептори гіпоталамусу Sodium receptors of the hypothalamus

Осморецептори печінки Osmoreceptors of the liver

Глюкорецептори гіпоталамусу Hypothalamus glucoreceptors

Осморецептори гіпоталамусу Osmoreceptors of the hypothalamus

Барорецептори дуги аорти Aortic arch baroreceptors

2 / 200
Хворому з ревматоїдним артритом тривалий час вводили гідрокортизон. У нього з’явилися гіперглікемія, поліурія, глюкозурія, спрага. Ці ускладнення лікування є наслідком активації такого процесу: A patient with rheumatoid arthritis was administered hydrocortisone for a long time. He developed hyperglycemia, polyuria, glucosuria, thirst. These treatment complications are the result of the activation of this process:

Ліполіз Lipolysis

Глікогеноліз Glycogenolysis

Гліколіз Glycolysis

Глюконеогенез Gluconeogenesis

Глікогенез Glycogenesis

3 / 200
Для зняття марення і галюцинацій у хворої на шизофренію лікар використав аміназин. Який механізм антипсихотичної дії препарату? The doctor used aminazine to relieve delusions and hallucinations in a schizophrenic patient. What is the mechanism of antipsychotic action of the drug?

Блокада адренергічних і дофамінергічних процесів в ЦНС Blockade of adrenergic and dopaminergic processes in the central nervous system

Інгібування зворотнього нейронального захоплення МАО Inhibition of MAO neuronal reuptake

Стимуляція адренергічних і дофамінергічних процесів в ЦНС Stimulation of adrenergic and dopaminergic processes in the central nervous system

Блокада холінергічних процесів в ЦНС Blockade of cholinergic processes in the central nervous system

Стимуляція холінергічних процесів в ЦНС Stimulation of cholinergic processes in the central nervous system

4 / 200
При підозрі на туберкульоз хворій дитині зробили пробу Манту. Через 24 години у місці введення алергену з’явились припухлість, гіперемія і болісність. Які основні компоненти визначають цю реакцію організму? On suspicion of tuberculosis, a sick child was given a Mantoux test. After 24 hours, at the swelling, hyperemia and soreness appeared after the introduction of the allergen. What are the main components that determine this reaction of the body?

Макрофаги, В-лімфоцити і моноцити Macrophages, B-lymphocytes and monocytes

Мононуклеари, Т-лімфоцити і лімфокіни Mononuclear cells, T-lymphocytes and lymphokines

Плазматичні клітини, Т-лімфоцити і лімфокіни Plasma cells, T-lymphocytes and lymphokines

Гранулоцити, Т-лімфоцити і IgG Granulocytes, T-lymphocytes and IgG

В-лімфоцити, IgM B-lymphocytes, IgM

5 / 200
Хворому перед операцією було введено дитилін (лістенон) і проведено інтубацію. Дефіцит якого ферменту в організмі хворого подовжує дію м’язового релаксанту? Before the operation, the patient was administered ditilin (Listenone) and intubated. The deficiency of which enzyme in the patient's body prolongs the effect of the muscle relaxant?

N-ацетилтрансфераза N-acetyltransferase

Карбангідраза Carbohydrase

Псевдохолінестераза Pseudocholinesterase

K-Аа-АтФ-аза K-Aa-AtP-aza

Сукцинатдегідрогеназа Succinate dehydrogenase

6 / 200
Хлопчик 12-ти років знаходиться у лікарні з підозрою на харчову токсикоінфекцію. При посіві фекалій хворого на середовище Ендо виросла велика кількість безбарвних колоній. Який мікроорганізм можна з найбільшою імовірністю ВИКЛЮЧИТИ з числа можливих збудників захворювання? A 12-year-old boy is in the hospital with suspected food poisoning. A large number of colorless colonies grew when the patient's feces were cultured on Endo's medium. Which microorganism is most likely EXCLUDE from the number of possible causative agents of the disease?

Proteus vulgaris Proteus vulgaris

Escherichia coli Escherichia coli

Yersinia enterocolitica Yersinia enterocolitica

Pseudomonas aeruginosa Pseudomonas aeruginosa

Salmonella enteritidis Salmonella enteritidis

7 / 200
У хворого внаслідок хронічного захворювання органів дихання, що супроводжується задишкою, тахікардією і ціанозом, при дослідженні газового складу крові виявлений розвиток гіпоксемії і гіперкапнії. Внаслідок якого з порушень зовнішнього дихання виникли ці зміни? The patient has a chronic respiratory disease accompanied by shortness of breath, tachycardia and cyanosis, the development of hypoxemia and hypercapnia was detected during the study of the gas composition of the blood. As a result of which external breathing disorders did these changes occur?

Гіпервентиляція Hyperventilation

Гіпердифузія Hyperdiffusion

Гіповентиляція Hypoventilation

Гіпоперфузія Hypoperfusion

Гіперперфузія Hyperperfusion

8 / 200
У тварини через 2 тижні після експериментального звуження ниркової артерії підвищився артеріальний тиск. З посиленням дії на судини якого чинника гуморальної регуляції це пов’язане? In an animal 2 weeks after experimental narrowing of the renal artery increased blood pressure. This is related to the strengthening of the effect on vessels of which factor of humoral regulation?

Дофамін Dopamine

Альдостерон Aldosterone

Вазопресин Vasopressin

Ангіотензин II Angiotensin II

Кортизол Cortisol

9 / 200
У чоловіка 43-х років, що помер у хірургічному відділенні при явищах розлитого гнійного перитоніту, на розтині у дистальному відділі тонкого кишечника виявлені пейєрові бляшки, що виступають у просвіт, поверхня деяких з них вкрита струпом, коричнево-зеленуватого кольору, у центрі деяких бляшок наявні глибокі дефекти, що сягають серозного шару. Для якого захворювання характерні зміни, виявлені у тонкому кишечнику? In a 43-year-old man who died in the surgical department due to diffuse purulent peritonitis, an autopsy revealed Peyer's patches protruding into the lumen in the distal part of the small intestine , the surface of some of them is covered with scabs, brownish-green in color, in the center of some plaques there are deep defects that reach the serous layer. What disease is characterized by the changes found in the small intestine?

Дизентерія Dysentery

Хвороба Крона Crohn's disease

Черевний тиф Typhoid

Стафілококовий ентерит Staphylococcal enteritis

Туберкульоз кишечника Intestinal tuberculosis

10 / 200
У хворого виявлено ожиріння, гірсутизм, «місяцеподібне» обличчя, рубці багряного кольору на шкірі стегон. Артеріальний тиск -180/110 мм рт. ст., глюкоза крові - 17,2ммоль/л. При якій зміні продукції гормонів наднирників можлива така картина? The patient was found to have obesity, hirsutism, a 'moon-shaped' face, scars of purple color on the skin of the thighs. Blood pressure -180/110 mm Hg. st., blood glucose - 17.2 mmol/l. At what change in the production of hormones of the adrenal glands is such a picture possible?

Гіпопродукція мінералокортикоїдів Hypoproduction of mineralocorticoids

Гіперпродукція глюкокортикоїдів Hyperproduction of glucocorticoids

Гіперпродукція мінералокортикоїдів Hyperproduction of mineralocorticoids

Гіпопродукція адреналіну Hypoproduction of adrenaline

Гіпопродукція глюкокортикоїдів Hypoproduction of glucocorticoids

11 / 200
У хворого, що тривало приймав глюкокортикоїди, в результаті відміни препарату виникло загострення наявного захворювання, зниження артеріального тиску, слабкість. Ці явища можна пов’язати з розвитком: A patient who took glucocorticoids for a long time, as a result of withdrawal of the drug, exacerbation of the existing disease, lowering of blood pressure, weakness. These phenomena can be associated with development:

Гіперпродукцією АКТГ Hyperproduction of ACTH

Звикання до препарату Addiction to the drug

Недостатності наднирників Adrenal insufficiency

Кумуляцією Cumulative

Сенсибілізацією Sensitization

12 / 200
Тварині, сенсибілізованій туберкуліном, внутрішньоочеревинно введений туберкулін. Через 24 години при лапаротомії виявлено венозну гіперемію та набряк очеревини. Умазках-відбитках з очеревини велика кількість лімфоцитів та моноцитів. Яке запалення має місце у тварини? Tuberculin was administered intraperitoneally to an animal sensitized to tuberculin. After 24 hours, venous hyperemia and peritoneal edema were revealed during laparotomy. Smears-imprints from the peritoneum contain a large number of lymphocytes and monocytes. What inflammation occurs in the animal?

Серозне Serious

Гнійне Purulent

Фібринозне Fibrinous

Асептичне Aseptic

Алергічне Allergic

13 / 200
У хворого спостерігаються біль голови , висока температура, озноб, кашель. З харкотиння виділили палички овоїдної форми з біполярним забарвленням, грам-негативні, у мазку з бульйонної культури розташовані ланцюжками, на агарі утворюються колонії R-форми. Це характерне для збудника такого захворювання: The patient has a headache, high fever, chills, cough. From sputum ovoid-shaped rods with bipolar color were isolated, gram-negative, arranged in chains in a broth culture smear, R-form colonies formed on agar. This is characteristic of the causative agent of such a disease:

Менінгококовий назофарингіт Meningococcal nasopharyngitis

Дифтерія Diphtheria

Стрептококова ангіна Streptococcal angina

Чума Чума

Туберкульоз Tuberculosis

14 / 200
До лікарні потрапили пацієнти зі скаргами: слабкість, болі в кишечнику, розлади травлення. Після дослідження фекалій були виявлені кулясті цисти з чотирма ядрами. Для якого найпростішого характерні такі цисти? Patients came to the hospital with complaints: weakness, pain in the intestines, disorders digestion. After examining the feces, spherical cysts with four nuclei were found. For which protozoa are these cysts typical?

Балантидій Balantidius

Кишкова трихомонада Intestinal trichomonas

Амеба дизентерійна Dysenteric amoeba

Амеба кишкова Intestinal amoeba

Амеба ротова Oral amoeba

15 / 200
У доношеного новонародженого спостерігається жовте забарвлення шкіри та слизових оболонок. Імовірною причиною цього стану може бути тимчасова нестача такого ферменту: A full-term newborn has yellow skin and mucous membranes shell The probable cause of this condition may be a temporary lack of such an enzyme:

Уридинтрансфераза Uridinetransferase

УДФ-глюкуронілтрансфераза UDF-glucuronyltransferase

Гемоксигеназа Heme Oxygenase

Гемсинтетаза Hemsynthetase

Білівердинредуктаза Biliverdin reductase

16 / 200
У немовляти спостерігаються епілептиформні судоми, викликані дефіцитом вітаміну В6. Це спричинено зменшенням у нервовій тканині гальмівного медіатора - гамма-аміномасляної кислоти. Активність якого ферменту знижена: The baby has epileptiform seizures caused by vitamin B6 deficiency. This is caused by a decrease in the nerve tissue of the inhibitory mediator - gamma-aminobutyric acid. The activity of which enzyme is reduced:

Глутаматсинтетаза Glutamate synthetase

Глутаматдегідрогеназа Glutamate dehydrogenase

Піридоксалькіназа Pyridoxal Kinase

Аланінамінотрансфераза Alanine aminotransferase

Глутаматдекарбоксилаза Glutamate decarboxylase

17 / 200
У молодої особи впродовж року прогресувала ниркова недостатність зі смертельним наслідком. При розтині тіла виявлені великі строкаті нирки з червоними дрібними краплинами в жовто-сірому корковому шарі. Гістологічно в клубочках виявлені «півмісяці» з проліферуючого нефротелія. Який найбільш імовірний діагноз? A young person developed kidney failure over the course of a year with fatal as a result An autopsy revealed large variegated kidneys with small red drops in a yellow-gray cortical layer. Histologically, 'crescents' from proliferating nephrothelium were found in the glomeruli. What is the most likely diagnosis?

Крововилив в нирки Kidney hemorrhage

Амілоїдоз Amyloidosis

Швидкопрогресуючий гломерулонефрит Rapidly progressive glomerulonephritis

Рак нирки Kidney cancer

Гнійний нефрит Pure jade

18 / 200
У хворого відсутній зір, але зіничний рефлекс реалізується нормально. Де може знаходитись зона пошкодження? The patient has no vision, but the pupillary reflex is realized normally. Where can is there a damage zone?

Зоровий перехрест Sight cross

Нижні горбики чотиригорбкового тіла Inferior tubercles of quadrituberous body

Зорова кора Visual cortex

Верхні горбики чотиригорбкового тіла Upper tubercles of quadrituberous body

Соматосенсорна кора Somatosensory Cortex

19 / 200
Хворий з 30-річним стажем роботи у шахті помер від легенево-серцевої недостатності, що наростала. На розтині легені збільшені у розмірах, щільної консистенції, на розрізі у них велика кількість вузликів розмірами з просяні зерна і більше, щільних, сіруватого і сірувато-чорного кольору, місцями вузлики зливаються у більш великі ділянки. Назвіть найбільш імовірне захворювання: A patient with 30 years of experience working in a mine died of pulmonary and cardiac failure, which was increasing On autopsy, the lungs are enlarged in size, of a dense consistency, on section they have a large number of nodules the size of a millet grain or more, dense, grayish and grayish-black in color, in places the nodules merge into larger areas. Name the most probable disease:

Бериліоз Berylliosis

Антрако-силікоз Anthraco-silicosis

Азбестоз Asbestosis

Алюміноз Aluminoz

Сидероз Siderosis

20 / 200
Для серодіагностики черевного тифу ставлять реакцію, при якій до різних розведень сироватки хворого добавляють дiагностикуми трьох видів мікроорганізмів і результат якої оцінюють за наявності осаду із склеєних бактерій. Ця реакція відома під назвою: For the serodiagnosis of typhoid fever, a reaction is used in which to different dilutions diagnostics of three types of microorganisms are added to the patient's serum, and the result of which is assessed by the presence of sediment from glued bacteria. This reaction is known as:

Відаля Vidal

Вассермана Wasserman

Райта Wright

Асколі Ascoli

Борде-Жангу Borde-Jangu

21 / 200
У хворого напад тахікардії. Які мембранні циторецептори кардіоміоцитів доцільно заблокувати, щоб припинити напад? The patient has an attack of tachycardia. Which membrane cytoreceptors of cardiomyocytes are appropriate block to stop the attack?

Альфа-адренорецептори Alpha Adrenoreceptors

М- та Н-холінорецептори M- and H-cholinoreceptors

Н-холінорецептори N-cholinoreceptors

М-холінорецептори M-cholinoreceptors

Бета-адренорецептори Beta Adrenoreceptors

22 / 200
У хворого на ЕКГ виявлено, що тривалість інтервалу RR дорівнює 1,5 с, частота серцевих скорочень - 40/хв. Що є водієм ритму серця? The patient's ECG revealed that the duration of the RR interval is 1.5 s, the heart rate contractions - 40/min. What is the heart rhythm driver?

Синусовий вузол Sine Node

Атріовентрикулярний вузол Atrioventricular node

Права ножка Гіса His right leg

Пучок Гіса His Bundle

Ліва ножка Гіса His Left Leg

23 / 200
У хворого за лабораторними даними визначили аскаридоз. Який засіб треба призначити? The patient was diagnosed with ascariasis according to laboratory data. What remedy should be prescribed?

Гентаміцин Gentamicin

Піперазин Piperazine

Мебендазол Мебендазол

Фуразолідон Furazolidone

Фенасал Fenasal

24 / 200
Хворий не може відвести від тулуба верхню кінцівку. Який м’яз НЕ ВИКОНУЄ свою функцію? The patient cannot move the upper limb away from the trunk. Which muscle does NOT perform its function?

Дельтоподібний м’яз Deltoid

Малий коловий м’яз Small orbicularis oculi

Великий коловий м’яз Great orbs

Підосний м’яз Plates muscle

Найширший м’яз спини Latisus dorsi

25 / 200
Пункцію сечового міхура через передню черевну стінку, не займаючи очеревину, можливо виконати: Bladder puncture through the anterior abdominal wall, without occupying the peritoneum, possible to execute:

Тільки у дітей Only in children

Тільки у жінок Only in women

Тільки у чоловіків Only in men

При пустому сечовому міхурі With an empty bladder

При наповненому сечовому міхурі With a full bladder

26 / 200
В судово-медичній експертизі широко використовується метод дактилоскопії, який оснований на тому, що сосочковий шар дерми визначає строго індивідуальний малюнок на поверхні шкіри. Яка тканина утворює цей шар дерми? Dactyloscopy method is widely used in forensic medical examination, which based on the fact that the papillary layer of the dermis determines a strictly individual pattern on the surface of the skin. What tissue forms this layer of the dermis?

Жирова тканина Adipose tissue

Пухка волокниста неоформлена сполучна тканина Loose fibrous unformed connective tissue

Ретикулярна тканина Reticular tissue

Щільна оформлена сполучна тканина Dense designed connective tissue

Щільна неоформлена сполучна тканина Dense unformed connective tissue

27 / 200
Важливою складовою частиною ниркового фільтраційного бар’єру є тришарова базальна мембрана, яка має спеціальну сітчасту будову її середнього електроннощільного шару. Де міститься ця базальна мембрана? An important component of the renal filtration barrier is the three-layer basal a membrane that has a special mesh structure of its middle electron-dense layer. Where is this basement membrane located?

Ниркове тільце Renal corpuscle

Проксимальні канальці Proximal tubules

Тонкі канальці Thin tubules

Дистальні прямі канальці Distal straight tubules

Капіляри перитубулярної капілярної сітки Capillaries of the peritubular capillary network

28 / 200
Чоловік 42-х років помер при явищах вираженої інтоксикації і дихальної недостатності. На розтині: тканина легень у всіх відділах строката, з множинними дрібновогнищевими крововиливами та вогнищами емфіземи. Гістологічно у легенях: геморагічна бронхопневмонія з абсцедуванням, у цитоплазмі клітин епітелію бронхів еозинофільні і базофільні включення. Діагностуйте виявлене на секції захворювання: A 42-year-old man died of severe intoxication and respiratory failure. At autopsy: lung tissue in all departments is variegated, with multiple small foci of hemorrhages and foci of emphysema. Histologically in the lungs: hemorrhagic bronchopneumonia with abscessation, eosinophilic and basophilic inclusions in the cytoplasm of bronchial epithelial cells. Diagnose the disease detected on the section:

Плевропневмонія Pleuropneumonia

Крупозна пневмонія Croup pneumonia

Грип Грип

Стафілококова бронхопневмонія Staphylococcal bronchopneumonia

Часткова пневмонія Partial pneumonia

29 / 200
В експерименті подразнюють скелетний м’яз серією електричних імпульсів. Який вид м’язового скорочення виникне, якщо кожний наступний імпульс припадає на період вкорочення поодинокого м’язового скорочення? In an experiment, skeletal muscle is stimulated with a series of electrical impulses. What kind of muscle contraction will occur if each subsequent impulse falls on the period of shortening of a single muscle contraction?

Суцільний тетанус Solid tetanus

Серія поодиноких скорочень Series of single abbreviations

Асинхронний тетанус Asynchronous tetanus

Контрактура м’яза M'yaz contracture

Зубчастий тетанус Serrated tetanus

30 / 200
У хворого, який тривалий час страждав на переміжну кульгавість, тканини пальців стопи сухі, чорного кольору, нагадують мумію. На невеликій відстані від почорнілої ділянки розташована двоколірна лінія (червоний колір прилягає до практично незмінених тканин, а біло-жовтий колір - до змінених тканин). Який вид некрозу у даного хворого? In a patient who suffered from intermittent lameness for a long time, the tissues of the toes dry, black, resemble a mummy. A two-color line is located at a short distance from the blackened area (the red color is adjacent to practically unchanged tissues, and the white-yellow color is to the changed tissues). What type of necrosis does this patient have?

Секвестр Sequestration

Пролежень Decubitus

Гангрена Gangrene

!нфаркт !нфаркт

Мацерація Maceration

31 / 200
У пацієнта, що звернувся до лікаря, спостерігається жовте забарвлення шкіри, сеча - темна, кал темно-жовтого кольору. Підвищення концентрації якої речовини буде спостерігатися в сироватці крові? A patient who has consulted a doctor has a yellow skin color, urine - dark, dark yellow stool. An increase in the concentration of which substance will be observed in the blood serum?

Вердоглобін Verdoglobin

Білівердин Biliverdin

Кон’югований білірубін Conjugated bilirubin

Вільний білірубін Free bilirubin

Мезобілірубін Mesobilirubin

32 / 200
До дерматолога звернулася пацієнтка із скаргами на екзематозне ураження шкіри рук, що з’являється після контакту з миючим засобом «Лотос» Використання гумових рукавичок запобігає цьому. Патологічна реакція шкіри зумовлена активацією: A patient came to the dermatologist with complaints of eczematous lesions of the skin of the hands, that appears after contact with the 'Lotus' detergent. The use of rubber gloves prevents this. The pathological reaction of the skin is due to the activation:

В -лімфоцитів B-lymphocytes

Т-лімфоцитів T-lymphocytes

Нейтрофілів Neutrophils

Моноцитів Monocytes

Базофілів Basophils

33 / 200
У хворого на тимому (пухлину вилочкової залози) спостерігається ціаноз, розширення підшкірної венозної сітки і набряк м’яких тканин обличчя, шиї, верхньої половини тулуба і верхніх кінцівок. Який венозний стовбур перетиснено пухлиною? A patient with thymus (tumor of the thymus gland) has cyanosis, enlargement subcutaneous venous network and swelling of soft tissues of the face, neck, upper half of the body and upper limbs. Which venous trunk is compressed by the tumor?

Зовнішня яремна вена External jugular vein

Внутрішня яремна вена Internal jugular vein

Передня яремна вена Anterior jugular vein

Підключична вена Subclavian vein

Верхня порожниста вена Superior vena cava

34 / 200
В результаті травми у чоловіка 47-ми років пошкоджені передні корінці спинного мозку. Відростки яких нейронів пошкоджені? As a result of an injury, a 47-year-old man has damaged the anterior roots of the spinal cord. The processes of which neurons are damaged?

Дендрити рухових і аксони ядер бокових стовпів Dendrites of motor and axons of nuclei of lateral columns

Аксони чутливих псевдоуніполярних Axons of sensitive pseudounipolar

Дендрити чутливих псевдоуніполярних Dendrites of sensitive pseudounipolar

Дендрити і аксони чутливих псевдоуні-полярних Dendrites and axons of sensitive pseudounipolar

Аксони нейронів рухових соматичних та вегетативних ядер Axons of neurons of motor somatic and vegetative nuclei

35 / 200
Під час розтину померлого 43-х років, що страждав на ІХС з розвитком інфаркту міокарда, патологоанатом виявив набряк легень. Які патологічні зміни могли зумовити набряк легень? During the autopsy of a 43-year-old deceased who suffered from coronary heart disease with the development of a heart attack myocardium, the pathologist found pulmonary edema. What pathological changes could cause pulmonary edema?

Стаз крові Blood stasis

Ішемія малого кола Small circle ischemia

Гостра правошлуночкова недостатність Acute right ventricular failure

Гостра лівошлуночкова недостатність Acute left ventricular failure

Гостре загальне малокрів’я Acute general anemia

36 / 200
У чоловіка 40-ка років ушкоджені міжпальцеві проміжки на ногах: шкіра мокне, відшаровується, з’явилися тріщини. При посіві зі шкрібу шкіри на середовище Сабуро виросли пухнасті колонії, білі зверху та зеленувато-жовті внизу. У мазках з верхньої частини колоній видно конідії у вигляді «дубинок» з 1-5 клітинами. Які ще органи найбільш імовірно може уразити цей збудник? A 40-year-old man has damaged interdigital spaces on his feet: the skin becomes wet, peeling off, cracks appeared. When inoculating from a skin scraping on Saburo's medium fluffy colonies, white above and greenish-yellow below, have grown. In smears from the upper part of the colonies, conidia in the form of 'clubs' with 1-5 cells are visible. What other organs are most likely to be affected by this pathogen?

Слизова статевих шляхів Mucosa of the genital tract

Волосся Hair

Підшкірна клітковина Subcutaneous tissue

Нігті Nails

Лімфатичні судини Lymphatic vessels

37 / 200
При ендоскопічному дослідженні у хворого з хронічним ентероколітом (запалення кишки) спостерігається відсутність специфічних структур рельєфа тонкої кишки. Які компоненти визначають особливості рельєфа слизової оболонки цього органу? During endoscopic examination of a patient with chronic enterocolitis (inflammation intestines) the absence of specific structures of the relief of the small intestine is observed. What components determine the peculiarities of the relief of the mucous membrane of this organ?

Косо-спіральні складки Oblique spiral folds

Поля, ворсинки Fields, hairs

Гаустри, ворсинки, крипти Gaustry, villi, krypti

Циркулярні складки, ворсинки та крипти Circular folds, villi and crypts

Поля, складки, ямки Margins, folds, dimples

38 / 200
Під час операції на головному мозку відмічено, що подразнення певних зон кори великих півкуль викликало у хворого і тактильні і температурні відчуття. На яку звивину діяли подразники? During brain surgery, it was noted that irritation of certain areas of the cortex of large hemispheres caused tactile and temperature sensations in the patient. On which gyrus did the stimuli act?

Поясна Wasted

Верхня латеральна Upper lateral

Постцентральна Postcentral

Прецентральна Precentral

Парагіпокампова Parahippocampal

39 / 200
Хворому проведено субтотальну субфасціальну резекцію щитоподібної залози. У післяопераційному періоді тривалий час зберігається охриплість голосу. Який нерв ушкоджено в ході операції? The patient underwent subtotal subfascial resection of the thyroid gland. In the hoarseness of the voice persists for a long time in the postoperative period. What nerve was damaged during the operation?

Під’язиковий нерв Hypoglossal nerve

Верхній гортанний нерв Superior laryngeal nerve

Зворотній гортанний нерв Reverse laryngeal nerve

Язиковий нерв Lingual nerve

Нижньощелепний нерв Mandibular nerve

40 / 200
В результат виснажуючої м’язової роботи у робочого значно зменшилась буферна ємність крові. Надходженням якої речовини у кров можна пояснити це явище? As a result of exhausting muscle work, the worker's buffer capacity has significantly decreased of blood This phenomenon can be explained by the entry of what substance into the blood?

Піруват Pyruvate

1,3-бісфосфогліцерат 1,3-bisphosphoglycerate

Альфа-кетоглутарат Alpha-Ketoglutarate

3-фосфоглицерат 3-phosphoglycerate

Лактат Лактат

41 / 200
В біоптаті бронха хворого, який зловживає палінням, в потовщеній слизовій оболонці виявлено хронічний обструктивний бронхіт з ознаками трансформації одношарового війчастого епітелія в багатошаровий плоский епітелій. Який з процесів найбільш імовірний? In the biopsy of a bronchus of a patient who abuses smoking, in the thickened mucous membrane chronic obstructive bronchitis with signs of single-layer transformation was detected ciliated epithelium into a stratified squamous epithelium. Which of the processes is the most likely?

Гіпертрофія епітелію Hypertrophy of the epithelium

Лейкоплакія Leukoplakia

Метаплазія Metaplasia

Плоскоклітинний рак Squamous cell carcinoma

Гіперплазія епітелію Epithelial hyperplasia

42 / 200
У хворого 45-ти років при аналізі ЕКГ встановлено: ритм синусовий, число передсердних комплексів більше числа шлуночкових комплексів; прогресуюче подовження інтервалу P-Q від комплексу до комплексу; випадіння окремих шлуночкових комплексів; зубці P та комплекси QRST без змін. Назвіть тип порушення серцевого ритму: In a 45-year-old patient, ECG analysis revealed: sinus rhythm, number of atrial complexes more than the number of ventricular complexes; progressive lengthening of the PQ interval from complex to complex; loss of individual ventricular complexes; P waves and QRST complexes are unchanged. Name the type of heart rhythm disorder:

Атріовентрикулярна блокада I ступеня Atrioventricular block I degree

Атріовентрикулярна блокада II ступеня Atrioventricular block II degree

Синоаурікулярна блокада Sino-auricular blockade

Внутрішньопередсердна блокада Intra-atrial blockade

Повна атріовентрикулярна блокада Complete atrioventricular block

43 / 200
Хворий з діагнозом цукровий діабет, вранці натще отримав призначену дозу інсуліну пролонгованої дії. Пропустив черговий прийом їжі і невдовзі відчув слабкість, біль голови ,запаморочення, пітливість, тремтіння тіла, судоми, відчуття голоду, явища гіпоглікемії. Застосування глюкози стан не полегшило. Який препарат необхідно ввести для купірування даного стану? A patient diagnosed with diabetes received the prescribed dose of insulin in the morning on an empty stomach prolonged action. I missed another meal and soon felt weakness, headache, dizziness, sweating, body tremors, convulsions, hunger, hypoglycemia. The use of glucose did not alleviate the condition. What drug should be administered to stop this condition?

Тріамцінолон Triamcinolone

Адреналін Adrenaline

Норадреналін Noradrenaline

Гідрокортизон Hydrocortisone

Пренізолон Prenisolone

44 / 200
У чоловіка 25-ти років з переломом основи черепа виділяється великий об’єм сечі з низькою відносною щільністю. Причиною змін сечоутворення є порушення синтезу такого гормону: A 25-year-old man with a skull base fracture has a large volume of urine with low relative density. The reason for changes in urine formation is a violation of the synthesis of such a hormone:

Тиреотропний гормон Thyrotropic hormone

Адренокортикотропний гормон Adrenocorticotropic Hormone

Соматотропний гормон Somatotropin

Вазопресин Vasopressin

Окситоцин Oxytocin

45 / 200
Розпочинається імплантація бластоцисти людини. Як називається період ембріогенезу, що розпочинається одночасно з імплантацією? Implantation of a human blastocyst begins. What is the period of embryogenesis called, what begins at the same time as implantation?

Гістогенез Histogenesis

Гаструляція Gastrulation

Диференціювання Differentiation

Інвагінація Intussusception

Дроблення Crush

46 / 200
Дитина 8-ми років готувалася до тонзилектомії. Аналіз крові показав, що час згортання збільшено (до 7 хвилин). Який препарат за 5 днів до операції слід включити до комплексу лікарських засобів підготовчого періоду в першу чергу? An 8-year-old child was being prepared for a tonsillectomy. A blood test showed that the clotting time increased (up to 7 minutes). Which drug 5 days before the operation should be included in the complex of drugs of the preparatory period in the first place?

Кальцію хлорид Calcium chloride

Дицинон Dicynone

Фібриноген Fibrinogen

Вікасол Вікасол

Амінокапронова кислота Aminocaproic acid

47 / 200
У дитини, що страждає на пілоростеноз, що супроводжується частим блюванням, розвинулись ознаки зневоднення організму. Яка форма порушення кислотно-основного стану може розвиватися у даному випадку? In a child suffering from pylorostenosis accompanied by frequent vomiting, signs of dehydration have developed. What form of acid-base disturbance can develop in this case?

Негазовий алкалоз Nongaseous alkalosis

Метаболічний ацидоз Metabolic acidosis

Газовий ацидоз Gas acidosis

Газовий алкалоз Gas alkalosis

Негазовий ацидоз Nongaseous acidosis

48 / 200
У хворого після автомобільної травми артеріальний тиск - 70/40 мм рт. ст. Хворий у непритомному стані. За добу виділяє близько 550 мл сечі. Яке порушення функції нирок спостерігається у хворого? The patient's blood pressure after a car injury is 70/40 mm Hg. The patient has in an unconscious state. It releases about 550 ml of urine per day. What impairment of kidney function is observed in the patient?

Пієлонефрит Pyelonephritis

Гострий дифузний гломерулонефрит Acute diffuse glomerulonephritis

Тубулопатія Tubulopathy

Хронічна ниркова недостатність Chronic renal failure

Гостра ниркова недостатність Acute renal failure

49 / 200
У гістопрепараті яєчника жінки визначаються структури, що мають велику порожнину. Овоцит І порядку в них оточений прозорою оболонкою, променистим вінцем і розташований у яйценосному горбику, стінка утворена шаром фолікулярних клітин i текою. Вкажіть, якій структурі яєчника належать дані морфологічні ознаки: Structures with a large cavity are identified in the histopreparation of a woman's ovary. In them, the oocyte of the first order is surrounded by a transparent shell, a radiant crown and is located in the egg-bearing tubercle, the wall is formed by a layer of follicular cells and theca. Indicate which structure of the ovary these morphological features belong to:

Атретичне тіло Atretic body

Примордіальний фолікул Primordial follicle

Жовте тіло Yellow body

Зрілий (третинний) фолікул Mature (tertiary) follicle

Первинний фолікул Primary follicle

50 / 200
Хворий переніс повторний інтрамуральний інфаркт міокарда. Після лікування та реабілітації виписаний у задовільному стані під нагляд дільничного терапевта. Через 2роки загинув у автомобільній катастрофі. Який характер патологічного процесу в міокарді було встановлено на розтині? The patient suffered a repeated intramural myocardial infarction. After treatment and he was discharged from rehabilitation in a satisfactory condition under the supervision of a district therapist. He died in a car accident 2 years later. What was the nature of the pathological process in the myocardium established at autopsy?

Гіперплазія Hyperplasia

Некроз Necrosis

Дрібновогнищевий кардіосклероз Microfocal cardiosclerosis

Крупновогнищевий кардіосклероз Large focal cardiosclerosis

Атрофія Atrophy

51 / 200
При бактеріологічному дослідженні гною з післяопераційної рани виділені мікроорганізми, які дали ріст на цукрово-кров’яному агарі через 7-10 днів в анаеробних умовах: колонії S-форми, блискучі, чорного кольору з неприємним запахом. При мікроскопії виявлені поліморфні грамнегативні палички. Які мікроорганізми могли викликати цей нагнійний процес? During the bacteriological examination of pus from the postoperative wound, microorganisms that gave growth on sugar-blood agar after 7-10 days in anaerobic conditions: S-shaped colonies, shiny, black in color with an unpleasant smell. Microscopy revealed polymorphic gram-negative rods. What microorganisms could cause this purulent process?

Бактероїди Bacteroids

Велонели Veloneli

Клостридії Clostridia

Кишкова паличка Escherichia coli

Фузобактерії Fusobacteria

52 / 200
У 12-річного хлопчика в сечі виявлено високий вміст усіх амінокислот аліфатичного ряду. При цьому відмічена найбільш висока екскреція цистину та цистеїну. Крім того, УЗД нирок показало наявність каменів у них. Виберіть можливу патологію: A 12-year-old boy had a high content of all amino acids of the aliphatic series in his urine. At the same time, the highest excretion of cystine and cysteine was noted. In addition, ultrasound of the kidneys showed the presence of stones in them. Choose a possible pathology:

Цистинурія Cystinuria

Цистит Cystitis

Фенілкетонурія Phenylketonuria

Алкаптонурія Alkaptonuria

Хвороба Хартнупа Hartnup's disease

53 / 200
Цикл Кребса відіграє важливу роль у реалізації глюкопластичного ефекту амінокислот. Це зумовлено обов’язковим перетворенням безазотистого залишку амінокислот у: The Krebs cycle plays an important role in realizing the glucoplastic effect of amino acids. This is due to the mandatory conversion of the nitrogen-free amino acid residue into:

Сукцинат Сукцинат

Оксалоацетат Oxaloacetate

Фумарат Fumarate

Цитрат Citrate

Малат Малат

54 / 200
Після прийому препарату у хворого з серцевою недостатністю зменшилася частота скорочень серця, пульс став кращого наповнення, зменшилися набряки, збільшився діурез. Вкажіть, який препарат приймав хворий: After taking the drug in a patient with heart failure, the frequency of contractions of the heart, the pulse became better filled, swelling decreased, diuresis increased. Indicate which drug the patient took:

Дилтіазем Diltiazem

Дигоксин Digoxin

Анаприлін Anaprilin

Резерпін Backup

Верапаміл Verapamil

55 / 200
У пацієнта 65-ти років з тривалими скаргами, характерними для хронічного гастриту, у периферичній крові виявлені мегалоцити, у кістковому мозку мегалобластичний еритропоез. Який найбільш імовірний діагноз? A 65-year-old patient with long-term complaints characteristic of chronic gastritis, in megaloblastic erythropoiesis was found in the peripheral blood and megaloblastic erythropoiesis in the bone marrow. What is the most likely diagnosis?

B12-фолієводефіцитна анемія B12-folate deficiency anemia

Залізодефіцитна анемія Iron deficiency anemia

Гіпопластична анемія Hypoplastic anemia

Гемолітична анемія Hemolytic anemia

Апластична анемія Aplastic anemia

56 / 200
З калу та блювотних мас від хворого з підозрою на холеру були виділені культури вібріонів. Проведення якої реакції дозволить визначити вид мікроба, що викликав це захворювання? Cultures were isolated from feces and vomitus from a patient with suspected cholera vibrios Conducting which reaction will determine the type of microbe that caused this disease?

Аглютинації з сироватками, що містять Н-антитіла Agglutinations with sera containing H-antibodies

Преципітації Precipitation

Пасивної гемаглютинації з еритроци-тарним антигенним діагностикумом Passive hemagglutination with erythrocyte antigen diagnostics

Аглютинації Відаля Vidal agglutination

Аглютинації з сироватками, що містять О-антитіла Agglutinations with sera containing O-antibodies

57 / 200
У дитини на слизовій оболонці щік та язика виявлені білуваті плями, які нагадують молоко, що скипілося. У виготовлених препаратах-мазках знайдені грампозитивні овальні дріжджоподібні клітини. Які це збудники? The child has whitish spots on the mucous membrane of the cheeks and tongue that resemble boiled milk. Gram-positive oval yeast-like cells were found in the prepared swab preparations. What are these pathogens?

Актиноміцети Actinomycetes

Гриби роду Кандіда Fungi of the genus Candida

Стафілококи Staphylococci

Дифтерійна паличка Diphtheria bacillus

Фузобактерії Fusobacteria

58 / 200
При обстеженні юнака з розумовою відсталістю виявлено євнухоїдну будову тіла, недорозвиненість статевих органів. В клітинах порожнини рота - статевий хроматин. Який метод генетичного дослідження слід застосувати для уточнення діагнозу? When examining a young man with mental retardation, a eunuchoid body structure was revealed, underdevelopment of the genitals. In the cells of the oral cavity - sex chromatin. What method of genetic research should be used to clarify the diagnosis?

Цитологічний Cytological

Клініко-генеалогічний Clinical-genealogical

Дерматогліфіка Dermatoglyphics

Популяційно-статистичний Population-statistical

Біохімічний Biochemical

59 / 200
У хворого з жовтяницею встановлено: підвищення у плазмі крові вмісту загального білірубіну за рахунок непрямого (вільного), в калі та сечі - високий вміст стеркобіліну, рівень прямого (зв’язаного) білірубіну в плазмі крові в межах норми. Про який вид жовтяниці можна думати? In a patient with jaundice, it was established: an increase in the blood plasma of total bilirubin at the expense of indirect (free), in feces and urine - a high content of stercobilin, the level of direct (bound) bilirubin in the blood plasma is within the normal range. What kind of jaundice can you think about?

Паренхіматозна (печінкова) Parenchymatous (hepatic)

Механічна Mechanical

Хвороба Жильбера Gilbert's disease

Гемолітична Hemolytic

Жовтяниця немовлят Infant Jaundice

60 / 200
Кухар в результаті необачності обпік руку парою. Підвищення концентрації якої речовини викликало почервоніння, набряклість та болючість ураженої ділянки шкіри? The cook carelessly burned his hand with steam. An increase in the concentration of which substance caused redness, swelling and soreness of the affected area of the skin?

Тіамін Thiamine

Лізин Lysine

Глутамін Glutamine

Гістамін Histamine

Галактозамін Galactosamine

61 / 200
Дитина доставлена в санпропускник в стані асфіксії. При огляді в гортані виявлені білуваті плівки, що обтурують просвіт та легко видаляються. Лікар запідозрив дифтерію. Про яке запалення гортані йдеться? The child was taken to the sanatorium in a state of asphyxiation. During the examination of the larynx, whitish films obscuring the lumen and easily removed. The doctor suspected diphtheria. What inflammation of the larynx is it about?

Крупозне Крупозне

Катаральне Cataral

Гнійне Purulent

Серозне Serious

Дифтеритичне Diphtheritic

62 / 200
У хворого із запаленням легень спостерігається підвищення температури тіла. Яка біологічно активна речовина відіграє провідну роль у виникненні цього прояву хвороби? A patient with pneumonia has an increase in body temperature. What does a biologically active substance play a leading role in the occurrence of this manifestation of the disease?

Інтерлейкін-I Interleukin-I

Серотонін Serotonin

Брадикінін Brady's

Лейкотрієни Leukotrienes

Гістамін Histamine

63 / 200
Хворий 12-ти років поступив в клініку з гемартрозом колінного суглоба, з раннього дитинства страждає кровоточивістю. Яка хвороба у хлопчика? A 12-year-old patient was admitted to the clinic with hemarthrosis of the knee joint, from early suffered from bleeding in childhood. What disease does the boy have?

B12 фолієво-дефіцитна анемія B12 folate deficiency anemia

Гемолітична анемія Hemolytic anemia

Геморагічний васкуліт Hemorrhagic vasculitis

Гемофілія Hemophilia

Тромбоцитопенічна пурпура Thrombocytopenic purpura

64 / 200
У хворого видалено 12-палу кишку. Це призведе до зменшення секреції, перш за все, такого гормону: The patient's duodenum has been removed. This will lead to a decrease in secretion, first of all, such a hormone:

Холецистокінін-секретин Cholecystokinin-secretin

Нейротензин Neurotensin

Гастрин Gastrin

Соматостатин Somatostatin

Гістамін Histamine

65 / 200
В клітині, яка мітотично ділиться, спостерігається розходження дочірніх хроматид до полюсів клітини. На якій стадії мітотичного циклу знаходиться клітина? In a mitotically dividing cell, daughter chromatids diverge to cell poles. At what stage of the mitotic cycle is the cell?

Анафаза Анафаза

Інтерфаза Interphase

Телофаза Telophase

Метафаза Metaphase

Профаза Профаза

66 / 200
Знешкодження ксенобіотиків (лікарських засобів, епоксидів, ареноксидів, альдегідів, нітропохідних тощо) та ендогенних метаболітів (естрадіолу, простагландинів, лейкотрієнів) відбувається в печінці шляхом їх кон’югації з: Disposal of xenobiotics (medicines, epoxides, arenoxides, aldehydes, nitroderivatives, etc.) and endogenous metabolites (estradiol, prostaglandins, leukotrienes) occurs in the liver through their conjugation with:

Фосфоаденозином Phosphoadenosine

Глутатіоном Glutathione

S-Аденозилметіоніном S-Adenosylmethionine

Гліцином Glycinema

Аспарагіновою кислотою Aspartic acid

67 / 200
Вітамін A у комплексі зі специфічними циторецепторами проникає через ядерні мембрани, індукує процеси транскрипції, що стимулює ріст та диференціювання клітин. Ця біологічна функція реалізується наступною формою вітаміну A: Vitamin A in complex with specific cytoreceptors penetrates through nuclear membranes, induces transcription processes that stimulate cell growth and differentiation. This biological function is realized by the following form of vitamin A:

Цис-ретиналь Cis Retinal

Транс -ретиналь Trans Retinal

Транс-ретиноєва кислота Trans Retinoic Acid

Ретинол Retinol

Каротин Carotene

68 / 200
Одна з форм вродженої патології супроводжується гальмуванням перетворення фенілаланіну в тирозин. Біохімічною ознакою хвороби є накопичення в організмі деяких органічних кислот, у тому числі такої кислоти: One of the forms of congenital pathology is accompanied by inhibition of transformation phenylalanine to tyrosine. A biochemical sign of the disease is the accumulation of some organic acids in the body, including the following acid:

Піровиноградна Pyrovyngradna

Фенілпіровиноградна Phenylpyruvate

Молочна Dairy

Лимонна Lemon

Ілутамінова Ilutaminova

69 / 200
Хворому встановлено діагноз - активний вогнищевий туберкульоз легень. Вкажіть, який із препаратів йому найбільш доцільно призначити в першу чергу The patient has been diagnosed with active focal pulmonary tuberculosis. Specify which of it is most expedient to prescribe drugs to him in the first place

Етіонамід Ethionamide

Ізоніазид Isoniazid

Циклосерин Cycloserine

Етоксид Etoxide

Сульфален Sulfalene

70 / 200
В експерименті на спинному мозку при збудженні альфа-мотонейронів згиначів встановлено гальмування альфа-мотонейронів м’язів-розгиначів. Який вид гальмування лежить в основі цього явища? In an experiment on the spinal cord during excitation of flexor alpha motoneurons inhibition of alpha motoneurons of extensor muscles was established. What type of inhibition is the basis of this phenomenon?

Деполяризаційне Depolarizing

Зворотнє Reverse

Латеральне Lateral

Пресинаптичне Presynaptic

Реципрокне Reciprocal

71 / 200
У хворого, що страждає на важку форму порушення водно-сольового обміну, настала зупинка серця у діастолі. Який найбільш імовірний механізм зупинки серця у діастолі? A patient suffering from a severe form of impaired water-salt metabolism has cardiac arrest in diastole. What is the most likely mechanism of cardiac arrest in diastole?

Гіперкаліємія Hyperkalemia

Дегідратація організма Dehydration of the body

Гіпокаліємія Hypokalemia

Гіпонатріємія Hyponatremia

Гіпернатріємія Hypernatremia

72 / 200
Встановлено ураження ВІЛ Т-лімфоцитів. При цьому фермент вірусу зворотня транскриптаза (РНК-залежна ДНК-полімераза) каталізує синтез: HIV damage to T-lymphocytes has been established. At the same time, the enzyme of the reverse virus transcriptase (RNA-dependent DNA polymerase) catalyzes the synthesis:

Вірусної РНК на матриці ДНК Viral RNA on a DNA matrix

ДНК на вірусній р-РНК DNA on viral p-RNA

Вірусної і-РНК на матриці ДНК Viral i-RNA on DNA matrix

ДНК на матриці вірусної і-РНК DNA on viral i-RNA matrix

і-РНК на матриці вірусного білку i-RNA on a viral protein matrix

73 / 200
Людина зробила спокійний видих. Як називається об’єм повітря, який міститься у неї в легенях при цьому? A person exhaled calmly. What is the volume of air contained in his lungs during this called?

Життєва ємність легень Vital lung capacity

Залишковий об’єм Remaining Volume

Дихальний об’єм Respiratory volume

Резервний об’єм видиху Reserve expiratory volume

Функціональна залишкова ємкість легень Functional residual lung capacity

74 / 200
Людина зробила максимально глибокий видих. Як називається об’єм повітря, що знаходиться в її легенях після цього? A person exhaled as deeply as possible. What is the volume of air that is in her lungs after that?

Залишковий об’єм Remaining Volume

Резервний об’єм видиху Exhalation reserve volume

Ємність вдиху Inhalation capacity

Альвеолярний об’єм Alveolar volume

Функціональна залишкова ємність легень Functional residual lung capacity

75 / 200
У людини необхідно оцінити стан клапанів серця. Яким з інструментальних методів дослідження доцільно скористатися для цього? A person needs to assess the condition of the heart valves. Which of the instrumental methods is it appropriate to use research for this?

Сфігмографія Sphygmography

Флебографія Phlebography

Зондування судин Vessel probing

Електрокардіографія Electrocardiography

Фонокардіографія Phonocardiography

76 / 200
Чоловіку 40-ка років за вимогою діагностичних тестів зробили лімфографію органів грудної порожнини. Хірург встановив, що пухлина вразила орган, з лімфатичних судин якого лімфа безпосередньо переходить в грудну протоку. Який це орган? A 40-year-old man underwent lymphography of the organs at the request of diagnostic tests chest cavity The surgeon established that the tumor affected the organ, from the lymphatic vessels of which the lymph directly passes into the thoracic duct. What organ is this?

Осердя Hearts

Стравохід Esophagus

Трахея Trachea

Лівий головний бронх Left main bronchus

Серце Heart

77 / 200
При гістологічному дослідженні біоптату шкіри виявлені гранульоми, які складаються з макрофагальних вузликів з наявністю лімфоцитів та плазматичних клітин. Крім того, зустрічаються великі макрофаги з жировими вакуолями, які містять запакованих у вигляді куль збудників захворювання (клітини Вірхова). Грануляційна тканина добре васкуляризована. Для якого захворювання характерна описана гранульома? The histological examination of a skin biopsy revealed granulomas consisting of macrophage nodules with the presence of lymphocytes and plasma cells. In addition, there are large macrophages with fat vacuoles, which contain pathogens packed in the form of balls (Virchow cells). The granulation tissue is well vascularized. For what disease is the described granuloma typical?

Риносклерома Rhinoscleroma

Лепра Лепра

Сап Сап

Туберкульоз Tuberculosis

Сифіліс Syphilis

78 / 200
При мікроскопічному дослідженні біоптату шкіри виявляються гранульоми, які складаються з епітеліоїдних клітин, оточених в основному Т-лімфоцитами. Серед епітеліоїдних клітин розташовуються поодинокі гігантські багатоядерні клітини типу Пирогова-Лангханса. В центрі гранульом виявляються ділянки казеозного некрозу. Кровоносні судини відсутні. Для якого захворювання характерні описані гранульоми? On microscopic examination of a skin biopsy, granulomas are revealed, which consist of epithelioid cells surrounded mainly by T-lymphocytes. Single giant multinucleated cells of the Pirogov-Langhans type are located among the epithelioid cells. Areas of caseous necrosis are found in the center of the granuloma. There are no blood vessels. For which disease are the described granulomas typical?

Риносклерома Rhinoscleroma

Туберкульоз Tuberculosis

Лепра Leprosy

Сап Сап

Сифіліс Syphilis

79 / 200
Ліквідатору наслідків аварії на Чорнобильські АЕС, що отримав велику дозу опромінення, проведено трансплантацію кісткового мозку. Через деякий час після проведеної операції у пацієнта діагностовано розвиток реакції трансплантат проти хазяїна. Які антигени послужили пусковим механізмом виникнення цієї реакції? To the liquidator of the consequences of the accident at the Chernobyl nuclear power plant, who received a large dose of radiation, a bone marrow transplant was performed. Some time after the operation, the patient was diagnosed with the development of a graft-versus-host reaction. What antigens served as a trigger for this reaction?

Антигени системи HLA клітин організму донора Antigens of the HLA system of cells of the donor's body

Антигени системи Rh еритроцитів ліквідатора Rh system antigens of liquidator erythrocytes

Антигенами системи ABO еритроцитів ліквідатора By antigens of the ABO system of erythrocytes of the liquidator

Антигени системи HLA клітин організму ліквідатора Antigens of the HLA system of the liquidator's body cells

Антигени HBs, HBc, Hbe Antigens HBs, HBc, Hbe

80 / 200
Під час гістологічного дослідження стулок мітрального клапана серця жінки 30-ти років було встановлено, що ендотеліальні клітини вогнищево десквамовані, в цих ділянках на поверхні стулки розташовані дрібні тромботичні нашарування, сполучна тканина стулки з явищами мукоїдного набухання з ділянками склерозу та васкуляризації. Діагностуйте вид клапанного ендокардиту: During a histological examination of the leaflets of the mitral valve heart of a 30-year-old woman it was established that the endothelial cells are focally desquamated, in these areas there are small thrombotic layers on the surface of the valve, the connective tissue of the valve with the phenomena of mucoid swelling with areas of sclerosis and vascularization. Diagnose the type of valvular endocarditis:

Гострий бородавчастий Sharp warty

Фібропластичний Fibroplastic

Поворотньо-бородавчастий Rotary-warty

Дифузний Diffuse

Поліпозно-виразковий Polypo-ulcerative

81 / 200
При органічних пошкодженнях головного мозку пам’ять може покращити: With organic brain damage, memory can improve:

Пірацетам Piracetam

Мезапам Мезапам

Кофеїн Caffeine

Діазепам Diazepam

Нітразепам Nitrazepam

82 / 200
У хворої під час травми стався розрив лобкового симфізу. Який тип з’єднання постраждав? The patient suffered a rupture of the pubic symphysis during the injury. What type of connection hurt?

Іеміартроз Iemiarthrosis

Синхондроз Synchondrosis

Синостоз Synostosis

Діартроз Diarthrosis

Синдесмоз Syndesmose

83 / 200
У хворого діагностовано інфаркт задньої частини міжшлуночкової перегородки. В ділянці якої кровоносної судини виникло порушення кровообігу? The patient was diagnosed with a heart attack of the posterior part of the interventricular septum. In the area which blood vessel did the blood circulation disorder occur?

R. interventricularis posterior R. interventricularis posterior

R. atrialis intermedius R. atrialis intermedius

R. marginalis sinister R. marginalis sinister

R. marginalis dexter R. marginalis dexter

R. circumflexus R. circumflexus

84 / 200
У дитячому відділенні інфекційної клініки хлопчику поставлено діагноз «дифтерія» Який препарат потрібно ввести хворому в першу чергу? In the children's department of the infectious disease clinic, the boy was diagnosed with 'diphtheria' What should the drug be administered to the patient first?

АДП АДП

АКДП АКДП

TABte TABte

Протидифтерійна антитоксична сироватка Anti-diphtheria anti-toxic serum

Дифтерійний анатоксин Diphtheria toxoid

85 / 200
У студента перед екзаменом виникла тахікардія. Які зміни на ЕКГ будуть свідчити про її наявність? The student developed tachycardia before the exam. What changes on the ECG will indicate its availability?

Розширення комплексу QRS QRS complex widening

Подовження сегменту Q-T Prolongation of the QT segment

Подовження інтервалу R-R RR interval extension

Подовження інтервалу P-Q PQ interval extension

Укорочення інтервалу R-R Shortening RR interval

86 / 200
При розтині тіла чоловіка, померлого від опікової хвороби, знайдено набряк головного мозку, збільшення печінки, а також нирок, кірковий шар яких широкий, блідо-сірий, мозковий - повнокровний. Мікроскопічно: некроз епітелію канальців головних відділів з деструкцією базальних мембран, набряк інтерстицію з лейкоцитарною інфільтрацією та крововиливами. Який з перелічених діагнозів найбільш імовірний? At the autopsy of the body of a man who died of a burn disease, swelling of the head was found brain, an increase in the liver, as well as the kidneys, the cortical layer of which is wide, pale gray, and the brain is full-blooded. Microscopically: necrosis of the epithelium of the tubules of the main sections with destruction of the basement membranes, interstitial edema with leukocyte infiltration and hemorrhages. Which of the listed diagnoses is the most probable?

Мієломна нирка Myeloma kidney

Тубулоінтерстиціальний нефрит Tubulointerstitial nephritis

Некротичний нефроз Necrotic nephrosis

Пієлонефрит Pyelonephritis

Подагрична нирка Gouty kidney

87 / 200
У хворої 59-ти років з нейроциркуляторною (первинною) артеріальною гіпотензією купіровано гіпотонічний криз підшкірним введенням 1 мл 20% розчину кофеїну бензоатунатрію. Механізм дії цього препарату полягає у тому, що він блокує: A 59-year-old patient with neurocirculatory (primary) arterial hypotension hypotonic crisis was stopped by subcutaneous injection of 1 ml of 20% caffeine benzoathunatium solution. The mechanism of action of this drug is that it blocks:

Альфа-адренорецептори Alpha Adrenoreceptors

Моноаміноксідазу Monoamine oxidase

Аденозинові рецептори Adenosine receptors

Бета-адренорецептори Beta Adrenoreceptors

Пуринові рецептори Purine receptors

88 / 200
Хворий похилого віку страждає на хронічний закреп, в основі якого лежить гіпотонія товстого кишечнику. Який препарат слід призначити хворому? An elderly patient suffers from chronic constipation, the basis of which is hypotension large intestine What drug should be prescribed to the patient?

Ацеклідин Aceclidine

Касторова олія Castor oil

Бісакодил Bisacodyl

Натрію сульфат Sodium sulfate

Прозерин Prozerin

89 / 200
У жінки через 6 місяців після пологів розвинулася маткова кровотеча. При гінекологічному огляді у порожнині матки виявлена тканина темно-червоного кольору змножинними порожнинами, що нагадує «губку» При мікроскопічному дослідженні пухлини у лакунах крові виявлені атипові світлі епітеліальні клітини Лангханса і гігантські клітини синцитіотрофобласту. Назвіть пухлину: A woman developed uterine bleeding 6 months after childbirth. a gynecological examination revealed dark red tissue with multiple cavities resembling a 'sponge' in the uterine cavity. Microscopic examination of the tumor revealed atypical light Langhans epithelial cells and giant syncytiotrophoblast cells in the blood lacunae. Name the tumor:

Хоріонепітеліома Chorioepithelioma

Плоскоклітинний незроговілий рак Squamous non-keratinous carcinoma

Міхурний занесок Bubble Drift

Аденокарцінома Adenocarcinoma

Фіброміома Fibroid

90 / 200
У хворого спостерігається гемералопія (куряча сліпота). Яка з перерахованих речовин володітиме лікувальною дією? The patient has hemeralopia (chicken blindness). Which of the listed substances will have a therapeutic effect?

Каротин Каротин

Карнітин Carnitine

Креатин Creatine

Карнозин Carnosine

Кератин Keratin

91 / 200
Хворий 18-ти років звернувся до лікарні із скаргами на шум та больові відчуття у вусі. Об’єктивно: у хворого гостре респіраторне захворювання, риніт. Крізь який отвір глотки інфекція потрапила до барабанної порожнини та викликала її запалення? An 18-year-old patient came to the hospital complaining of noise and pain in the ear. Objectively: the patient has an acute respiratory disease, rhinitis. Through which opening of the pharynx did the infection enter the tympanic cavity and cause its inflammation?

Глотковий отвір слухової труби Pharyngeal opening of the auditory tube

Хоани Хоани

Зів Ziv

Барабанний отвір слухової труби Tympanic opening of the auditory tube

Вхід до гортані Entrance to the larynx

92 / 200
Хворий 50-ти років страждає на гіпертонічну хворобу. Під час фізичного навантаження у нього з’явилося відчуття слабкості, нестачі повітря, синюшність слизової оболонки губ, шкіри обличчя. Дихання супроводжувалося відчутними на відстані вологими хрипами. Який механізм лежить в основі виникнення такого синдрому? A 50-year-old patient suffers from hypertension. During physical exertion in he had a feeling of weakness, lack of air, bluishness of the mucous membrane of the lips, facial skin. Breathing was accompanied by moist wheezes perceptible in the distance. What mechanism underlies the occurrence of such a syndrome?

Гостра лівошлуночкова недостатність Acute left ventricular failure

Хронічна лівошлуночкова недостатність Chronic left ventricular failure

Колапс Collapse

Хронічна правошлуночкова недостатність Chronic right ventricular failure

Тампонада серця Cardiac tamponade

93 / 200
Внаслідок тривалого голодування в організмі людини швидко зникають резерви вуглеводів. Який з процесів метаболізму за цих умов поновлює вміст глюкози в крові? As a result of prolonged starvation in the human body, reserves quickly disappear carbohydrates Which of the metabolic processes restores the glucose content in the blood under these conditions?

Анаеробний гліколіз Anaerobic glycolysis

Аеробний гліколіз Aerobic glycolysis

Глікогеноліз Glycogenolysis

Глюконеогенез Gluconeogenesis

Пентозофосфатний шлях Pentose phosphate pathway

94 / 200
У чоловіка 40-ка років було встановлено діагноз: серпоподібноклітинна анемія. Який механізм приводить до зменшення кількості еритроцитів в крові у цього хворого? A 40-year-old man was diagnosed with sickle cell anemia. What mechanism leads to a decrease in the number of erythrocytes in the blood of this patient?

Внутрішньосудинний гемоліз Intravascular hemolysis

Нестача вітаміну B12 і фолієвої кислоти Vitamin B12 and folic acid deficiency

Позасудинний гемоліз Extravascular hemolysis

Нестача білка Protein deficiency

Нестача заліза в організмі Lack of iron in the body

95 / 200
У жінки 52-х років при обстеженні було виявлено зниження кількості еритроцитів у крові та підвищення рівня вільного гемоглобіну в плазмі крові (гемоглобінемія). КП- 0,85. Який вид анемії спостерігається у хворої? In a 52-year-old woman, a decrease in the number of erythrocytes in the blood was detected during the examination and an increase in the level of free hemoglobin in the blood plasma (hemoglobinemia). CP - 0.85. What type of anemia is observed in the patient?

Набута гемолітична Acquired hemolytic

Анемія внаслідок порушення еритропоезу Anemia due to impaired erythropoiesis

Спадкова гемолітична Hereditary hemolytic

Хронічна постгеморагічна Chronic posthemorrhagic

Іостра постгеморагічна Iostra posthemorrhagic

96 / 200
Лікування туберкульозу здійснюється за допомогою комбінованої хіміотерапії, що поєднує речовини різного механізму дії. Який з протитуберкульозних засобів пригнічує транскрипцію ДНК в РНК мікобактерій? Tuberculosis is treated with combined chemotherapy, which combines substances with different mechanisms of action. Which of the antituberculosis drugs suppresses the transcription of DNA into RNA of mycobacteria?

ПАСК ПАСК

Рифампіцин Rifampicin

Етіонамід Ethionamide

Ізоніазид Isoniazid

Стрептоміцин Streptomycin

97 / 200
На тканину діють електричним імпульсом катодного напрямку, амплітуда якого дорівнює 70% порогу. Які зміни мембранного потенціалу клітин це викличе? The tissue is acted upon by an electric pulse of the cathodic direction, the amplitude of which is 70% of the threshold. What changes in the membrane potential of cells will this cause?

- -

Часткова деполяризація Partial depolarization

Гіперполяризація Hyperpolarization

Потенціал дії Action potential

Змін не буде There will be no changes

98 / 200
У людини визначили величину енерговитрат. У якому стані знаходилась людина, якщо її енерговитрати виявилися меншими за основний обмін? A person's energy expenditure was determined. In what state was the person, if its energy consumption turned out to be less than the main exchange?

Сон Сон

Легка робота Easy work

Відпочинок Vacation

Нервове напруження Nervous tension

Спокій Peace

99 / 200
При визначенні основного обміну з’ясовано, що його величина у досліджуваного менша за належну величину на 7%. Це означає, що інтенсивність процесів енергетичного метаболізму у досліджуваного: When determining the main exchange, it was found that its value in the researched is less for the proper value by 7%. This means that the intensity of energy metabolism processes in the subject:

Нормальна Normal

Помірно підвищена Moderately increased

Помірно знижена Moderately reduced

Суттєво знижена Significantly reduced

Суттєво підвищена Significantly increased

100 / 200
Внаслідок руйнування певних структур стовбуру мозку тварина втратила орієнтувальні рефлекси. Які структури було зруйновано? Due to the destruction of certain structures of the brain stem, the animal lost its orientation reflexes What structures were destroyed?

Червоні ядра Red kernels

Чорна речовина Black matter

Чотиригорбкові тіла Four humped bodies

Медіальні ядра ретикулярної формації Medial nuclei of the reticular formation

Вестибулярні ядра Vestibular nuclei

101 / 200
У новонародженого малюка педіатр виявив, що отвір крайньої плоті за величиною не перевищує діаметр сечовивідного каналу і голівка статевого члена не може виходити через такий отвір. Як називається цей стан? In a newborn baby, the pediatrician discovered that the opening of the foreskin was not exceeds the diameter of the urethra and the head of the penis cannot exit through such an opening. What is the name of this state?

Парафімоз Paraphimosis

Гермафродитизм Hermaphrodite

Фімоз Фімоз

Гіпоспадія Hypospadia

Епіспадія Epispadia

102 / 200
Психологічне дослідження встановило: у людини добра здатність швидко пристосовуватися до нового оточення, добра пам’ять, емоційна стійкість, висока працездатність. Найімовірніше, ця людина: Psychological research established that a good person has the ability to quickly adapt to a new environment, good memory, emotional stability, high efficiency. Most likely, this person:

Холерик Choleric

Флегматик Phlegmatic

Флегматик з елементами меланхоліка Phlegmatic with elements of melancholic

Меланхолік Melancholic

Сангвінік Sanguine

103 / 200
Хворий на ішемічну хворобу серця не повідомив лікаря, що в нього бувають напади бронхоспазму. Лікар призначив препарат, після прийому якого напади стенокардії стали рідше, але напади бронхоспазму стали частіше. Який препарат був призначений? A patient with coronary heart disease did not tell the doctor that he had seizures bronchospasm The doctor prescribed a drug, after taking which angina attacks became less frequent, but bronchospasm attacks became more frequent. What drug was prescribed?

Нітрогліцерин Nitroglycerin

Нітросорбід Nitrosorbide

Анаприлін Анаприлін

Верапаміл Verapamil

Дилтіазем Diltiazem

104 / 200
До ендокринолога звернулась хвора 45-ти років із скаргами на підвищення апетиту, сухість слизових оболонок ротової порожнини, збільшення діурезу. При обстеженні вперше виявлено інсулін незалежний діабет. Який з названих препаратів доцільно призначити хворій? A 45-year-old patient came to the endocrinologist with complaints of increased appetite, dryness of the mucous membranes of the oral cavity, increased diuresis. During the examination, insulin-independent diabetes was detected for the first time. Which of the named drugs should be prescribed to the patient?

Глібенкламід Glibenclamide

Вазопресин Vasopressin

Окситоцин Oxytocin

Інсулін Insulin

Адіурекрин Adiurecrin

105 / 200
Дитина 7-ми років поступила в інфекційне відділення зі скаргами на різкий біль у горлі, утруднення під час ковтання, підвищення температури тіла до 39oC, набряк шиї. Об’єктивно: мигдалики збільшені, їх слизова оболонка повнокровна, вкрита великою кількістю плівок білувато-жовтого кольору, які щільно прилягають до слизової оболонки. При спробі зняти плівку залишається глибокий дефект, який кровоточить. Який вид запалення має місце? A 7-year-old child was admitted to the infectious department with complaints of a sharp sore throat, difficulty in swallowing, increase in body temperature up to 39oC, swelling of the neck. Objectively: the tonsils are enlarged, their mucous membrane is full of blood, covered with a large number of whitish-yellow films, which tightly adhere to the mucous membrane. When you try to remove the film, a deep defect remains that bleeds. What kind of inflammation is taking place?

Дифтеритичне Diphtheritic

Крупозне Coarse

Геморагічне Hemorrhagic

Гнійне Purulent

Серозне Serious

106 / 200
У хворого на хронічний дифузний гломерулонефрит розвинулася хронічна недостатність нирок. В термінальній стадії ХНН розвивається оліго- та анурія, що спричиняється: A patient with chronic diffuse glomerulonephritis developed chronic insufficiency kidney In the terminal stage of CRF, oligo- and anuria develops, which is caused by:

Ішемією коркової речовини нирок внаслідок спазму судин Ischemia of the cortical substance of the kidneys due to spasm of blood vessels

Збільшенням реабсорбції води в дистальних канальцях Increased water reabsorption in distal tubules

Дисемінованим внутрішньосудинним зсіданням крові Disseminated intravascular coagulation

Зниженням маси діючих нефронів Decreasing the mass of functioning nephrons

Зменшенням фільтраційного тиску та фільтрації Decreasing filtration pressure and filtration

107 / 200
При розтині тіла померлого чоловіка 48-ми років в ділянці 1-го сегменту правої легені виявлено круглий утвір діаметром 5 см з чіткими контурами, оточений тонким прошарком сполучної тканини, виповнений білими крихкими масами. Діагностуйте форму вторинного туберкульозу: At the autopsy of the body of a deceased 48-year-old man in the area of the 1st segment of the right lung a round formation with a diameter of 5 cm with clear contours was found, surrounded by a thin layer of connective tissue, filled with white fragile masses. Diagnose the form of secondary tuberculosis:

Туберкулома Tuberculoma

Фіброзно-кавернозний туберкульоз Fibro-cavernous tuberculosis

Гострий кавернозний туберкульоз Acute cavernous tuberculosis

Гострий вогнищевий туберкульоз Acute focal tuberculosis

Казеозна пневмонія Case pneumonia

108 / 200
На препарат представлено орган, вкритий сполучнотканинною капсулою, від якої відходять трабекули. В органі можна розрізнити кіркову речовину, де містяться лiмфатичнi вузлики та мозкову речовину, представлену тяжами лімфоїдних клітин. Який орган представлений на препараті? The preparation presents an organ covered with a connective tissue capsule, from which trabeculae depart. In the organ, it is possible to distinguish the cortical substance, which contains lymph nodes, and the medulla, represented by strands of lymphoid cells. What organ is represented on the drug?

Тимус Thymus

Мигдалики Tonsils

Червоний кістковий мозок Red bone marrow

Селезінка Spleen

Лімфатичний вузол Lymph node

109 / 200
В регуляції фізіологічних функцій беруть участь іони металів. Один із них отримав назву «король месенджерів» Таким біоелементом посередником є: Metal ions are involved in the regulation of physiological functions. One of them was named 'the king of messengers' Such a bioelement as a mediator is:

Са++ Са++

K+ K+

Zn++ Zn++

Fe+++ Fe+++

Na+ Na+

110 / 200
Хворий звернувся до лікаря зі скаргами на болі в кульшовому суглобі та у всіх привідних м’язах і шкірі над ними при рухах. Який нерв ушкоджено? The patient turned to the doctor with complaints of pain in the hip joint and in all adductors muscles and the skin above them during movements. What nerve is damaged?

Стегновий нерв Femoral nerve

Затульний нерв Pulse nerve

Клубово-пахвинний нерв Club-Inguinal Nerve

Задній шкірний нерв стегна Posterior cutaneous nerve of thigh

Латеральний шкірний нерв стегна Lateral cutaneous nerve of thigh

111 / 200
Хворому 35-ти років для обстеження очного дна був призначений атропіну сульфат у вигляді очних крапель. Для відновлення акомодації йому закрапали пілокарпіну гідрохлорид, але це не дало бажаного ефекту. Що є причиною відсутності ефекту? A 35-year-old patient was prescribed atropine sulfate in in the form of eye drops. He was given pilocarpine hydrochloride as an instillation to restore accommodation, but this did not have the desired effect. What is the reason for the lack of effect?

Двосторонній антагонізм Bilateral antagonism

Синергізм Synergism

Звикання Addiction

Односторонній антагонізм One-sided antagonism

Тахіфілаксія Tachyphylaxis

112 / 200
В поліклініку до лікаря звернулася жінка 32-х років зі скаргами на відсутність в неї лактації після народження дитини. Дефіцитом якого гормону, найбільш імовірно, можна пояснити дане порушення? A 32-year-old woman came to the doctor at the polyclinic with complaints about her lack of lactation after the birth of a child. The deficiency of which hormone can most likely explain this disorder?

Пролактин Prolactin

Вазопресин Vasopressin

Глюкагон Glucagon

Соматотропін Somatotropin

Тиреокальцитонін Thyrocalcitonin

113 / 200
У хворого пухлина черевної порожнини, що стискає нижню порожнисту вену. Який кава- кавальний анастомоз на передній стінці живота забезпечить відтік венозної крові? The patient has an abdominal tumor compressing the inferior vena cava. What cava- will the caval anastomosis on the front wall of the abdomen ensure the outflow of venous blood?

Між хребетними і пупковими венами Between the vertebral and umbilical veins

Між непарною і напівнепарною венами Between odd and semi-odd veins

Між верхньою і нижньою надчеревними венами Between the superior and inferior epigastric veins

Між пупковою і верхньою надчеревною венами Between the umbilical and superior epigastric veins

Між верхніми і нижніми прямокишковими венами Between the superior and inferior rectal veins

114 / 200
У людини під дією мутагенного фактору з’явилась велика кількість мутантних клітин. Але більшість з них були розпізнані і знищені клітинами: A large number of mutant cells appeared in a person under the influence of a mutagenic factor. But most of them were recognized and destroyed by cells:

Т-лімфоцитами кілерами Killer T-lymphocytes

Стовбуровими Stemmed

Т-лімфоцитами супресорами by suppressor T-lymphocytes

В -лімфоцитами By lymphocytes

Плазмобластами Plasmoblasts

115 / 200
При пошкодженні клітини іонізуючим випромінюванням вмикаються механізми захисту і адаптації. Який механізм відновлення порушеного внутрішньоклітинного гомеостазу реалізується при цьому? When the cell is damaged by ionizing radiation, protection mechanisms are activated and adaptation What mechanism of restoration of disturbed intracellular homeostasis is implemented in this case?

Пригнічення аденілатциклази Inhibition of adenylate cyclase

Накопичення Na+ в клітинах Na+ accumulation in cells

Активація Са-опосередкованих клітинних функцій Activation of Ca-mediated cellular functions

Гіпертрофія мітохондрій Mitochondrial hypertrophy

Активація антиоксидантної системи Activation of the antioxidant system

116 / 200
На гістологічному препараті легень видно структуру діаметром близько 0,5 мм, слизова оболонка якої вкрита одношаровим кубічним війковим епітелієм, у якому зустрічаються секреторні клітини Клара, війчасті клітини, мікроворсинчасті. Вкажіть структуру: On the histological preparation of the lungs, a structure with a diameter of about 0.5 mm is visible, mucous the shell of which is covered with a single-layered cubic ciliated epithelium, in which there are Clara secretory cells, ciliated cells, and microvillous cells. Specify the structure:

Термінальна бронхіола Terminal bronchiole

Середній бронх Medium bronchus

Альвеола Alveola

Альвеолярний хід Alveolar course

Малий бронх Small bronchus

117 / 200
Чоловік 36-ти років, лісник за фахом, через тиждень після тривалого перебування у весняному лісі гостро захворів -гарячка, головний біль, порушення свідомості, епілептиформні напади, смерть розвинулася на 3-й день хвороби. На роз- тині тіла: набряк головного мозку, множинні точкові геморагії; під час мікроскопічного дослідження - периваскулярний та перицелюлярний набряк, множинні периваскулярні,переважно лімфоцитарні, інфільтрати. Діагностуйте основне захворювання: A 36-year-old man, a forester by profession, a week after a long stay in In the spring forest, he became acutely ill - fever, headache, loss of consciousness, epileptiform seizures, death occurred on the 3rd day of the illness. On autopsy: cerebral edema, multiple point hemorrhages; during microscopic examination - perivascular and pericellular edema, multiple perivascular, mainly lymphocytic, infiltrates. Diagnose the main disease:

Менінгококова інфекція Meningococcal infection

Поліомієліт Polio

Кліщовий енцефаліт Tick-borne encephalitis

Гнійний енцефаліт Suppurative encephalitis

Церебро-васкулярна хвороба Cerebrovascular disease

118 / 200
У хворого з флегмоною передпліччя при мікробіологічному аналізі ексудату в зоні запалення визначена присутність стрептококів. Які клітини будуть переважати в ексудаті? In a patient with phlegmon of the forearm during microbiological analysis of the exudate in the area inflammation determined the presence of streptococci. What cells will prevail in exudates?

Базофільні гранулоцити Basophilic granulocytes

Нейтрофільні гранулоцити Neutrophil granulocytes

Моноцити Monocytes

Лімфоцити Lymphocytes

Еозинофільні гранулоцити Eosinophilic granulocytes

119 / 200
У п’ятимісячної дівчинки виявлено застійні явища у легенях. При обстеженні виявлено зв’язок між висхідною аортою та легеневою артерією, що в нормі спостерігається у деяких земноводних і плазунів. Назвіть цю природжену ваду розвитку: A five-month-old girl was found to have congestion in the lungs. During the examination, it was found the connection between the ascending aorta and the pulmonary artery, which is normally observed in some amphibians and reptiles. Name this congenital malformation:

Розвиток правої дуги аорти Development of the right aortic arch

Дефект міжпередсердної перегородки Atrial septal defect

Дефект міжшлуночкової перегородки Ventricular septal defect

Незрощення боталової протоки Nonunion of the botal duct

Транспозиція магістральних судин Transposition of trunk vessels

120 / 200
Хворому на гострий інфаркт міокарда у комплексній терапії було призначено гепарин. Через деякий час після введення даного препарату з’явилася гематурія. Який антагоніст гепарину необхідно ввести хворому для усунення даного ускладнення? A patient with an acute myocardial infarction was prescribed heparin in complex therapy. Some time after the introduction of this drug, hematuria appeared. What heparin antagonist must be administered to the patient to eliminate this complication?

Неодикумарин Neocoumarin

Протаміну сульфат Protamine Sulfate

Фібриноген Fibrinogen

Амінокапронова кислота Aminocaproic acid

Вікасол Vikasol

121 / 200
До лікарні звернулася жінка з дитиною, у якої на голові була гангренозна рана. Лікар при огляді виявив у рані білих червоподібних личинок комах. Яка комаха могла їх відкласти? A woman with a child who had a gangrenous wound on her head came to the hospital. The doctor at examination revealed white worm-like insect larvae in the wound. What insect could lay them down?

Муха-жигалка Stinger Fly

Москіти Mosquitoes

Комарі Mosquitoes

Вольфартова муха Wolfarth Fly

Блохи Fleas

122 / 200
На дослідження в бактеріологічну лабораторію було відправлено випорожнення хворої дитини грудного віку, з яких виділена культура ентеропатогенних кишкових паличок О55К59. На основі яких критеріїв виділена культура віднесена до ЕПКП О55? The stool of the patient was sent to the bacteriological laboratory for examination infants, from which a culture of enteropathogenic E. coli O55K59 was isolated. On the basis of what criteria is the selected culture assigned to EPKP O55?

Біохімічні властивості Biochemical properties

Морфологічні ознаки Morphological signs

Антигенні властивості Antigenic properties

Визначення фаговару Phagovar definition

Культуральні ознаки Cultural Features

123 / 200
У чоловіка 30-ти років перед операцією визначили групову належність крові. Кров резус- позитивна. Реакцію аглютинації еритроцитів не викликали стандартні сироватки груп 0а/3 (I), А/3 (II), Ва (III). Досліджувана кров належить до групи: The blood group of a 30-year-old man was determined before the operation. Blood Rh- positive Standard sera of groups 0a/3 (I), A/3 (II), Ba (III) did not cause an agglutination reaction of erythrocytes. The studied blood belongs to the group:

- -

Ар (II) Ар (II)

0ав (I) 0ав (I)

Ва (III) Ва (III)

АВ (IV) АВ (IV)

124 / 200
Швидкість проведення збудження нервовими волокнами становить 120 м/с. Який з наведених чинників, перш за все, забезпечує таку швидкість? The speed of conduction of excitation by nerve fibers is 120 m/s. Which of of the above factors, above all, provides such a speed?

Наявність мієлінової оболонки Presence of myelin sheath

Великий потенціал спокою Great rest potential

Малий поріг деполяризації Small depolarization threshold

Велика амплітуда потенціалу дії Large action potential amplitude

Великий фактор надійності Large reliability factor

125 / 200
У тварини зруйнували отолітові вестибулорецептори. Які з наведених рефлексів зникнуть внаслідок цього у тварини? The animal's otolith vestibuloreceptors were destroyed. Which of the following reflexes will disappear as a result of this in the animal?

Первинні орієнтувальні Primary indicative

Випрямлення тулуба Torso straightening

Статокінетичні при рухах з лінійним прискоренням Statokinetic when moving with linear acceleration

Статокінетичні при рухах з кутовим прискоренням Statokinetic during movements with angular acceleration

Міотатичні Myotatic

126 / 200
У хворого з ВШ-інфекцією визначена ураження шкіри нижніх кінцівок у вигляді множинних пухлинних вузликів синюшно-червоного кольору, що зливаються і створюють поверхневі виразки. При дослідженні біоптату шкіри виявлено новоутворення кровоносних судин, що створюють порожнини різної форми і величини та побудоване з ендотелію, багато пучків веретеноподібних клітин. Діагностуйте найімовірніший характер шкірної патології: In a patient with HS infection, a skin lesion of the lower extremities was determined in the form of multiple bluish-red tumor nodules that merge and create surface ulcers. During the examination of the skin biopsy, a neoplasm of blood vessels was found, creating cavities of different shapes and sizes and built from endothelium, many bundles of spindle-shaped cells. Diagnose the most likely nature of the skin pathology:

Саркома Капоші Kaposi's sarcoma

Лімфома шкіри Skin lymphoma

Базаліома Basal tumor

Дерматомікоз Dermatomycosis

Запальний дерматит Inflammatory dermatitis

127 / 200
У хворого з патологією серцево-судинної системи розвинулись набряки на нижніх кінцівках. Який механізм розвитку серцевого набряку? A patient with a pathology of the cardiovascular system developed swelling on the lower limbs What is the mechanism of development of cardiac edema?

Підвищення гідростатичного тиску в венулах Increased hydrostatic pressure in venules

Підвищення гідростатичного тиску в артеріолах Increased hydrostatic pressure in arterioles

Порушення лімфовідтоку Disruption of lymphatic drainage

Підвищення онкотичного тиску плазми крові Increased blood plasma oncotic pressure

Зниження осмотичного тиску плазми крові Decrease in blood plasma osmotic pressure

128 / 200
У хворої при лікуванні гіпертонічної хвороби на фоні прийому препарату виник сухий кашель. Для якого препарату характерна така побічна дія? During the treatment of hypertension, the patient developed a dry cough. Which drug is characterized by such a side effect?

Но-шпа No-shpa

Резерпін Reserpine

Лізиноприл Lisinopril

Гідрохлортіазид Hydrochlorothiazide

Дротаверин Drotaverine

129 / 200
У фібробластах шкіри дитини із хворобою Дауна виявлено 47 хромосом. Визначте тип аномалії: In fibroblasts of the skin of a child with Down's disease, 47 chromosomes were found. Determine the type anomalies:

Полісомія Y Polysomy Y

Трисомія Х Trisomia X

Трисомія 13 Trisomy 13

Трисомія 21 Trisomy 21

Трисомія 18 Trisomy 18

130 / 200
При операції у хлопчика 12-ти років видалений апендикс, який надіслано патологу на дослідження. Макроскопічно: апендикс в дистальному відділі з булавоподібним стовщенням діаметром 3 см, при розрізі якого вилилася прозора жовтувата рідина, стінка апендикса стоншена. Мікроскопічно: атрофія всіх шарів апендикса, ознак запалення немає. Який найбільш імовірний діагноз? During the operation of a 12-year-old boy, the appendix was removed, which was sent to the pathologist for research. Macroscopically: appendix in the distal part with a club-like thickening with a diameter of 3 cm, when cut, a transparent yellowish liquid poured out, the wall of the appendix is thinned. Microscopically: atrophy of all layers of the appendix, no signs of inflammation. What is the most likely diagnosis?

Міксоглобульоз апендикса Myxoglobulosis of the appendix

Емпієма апендикса Empyema of the appendix

Флегмонозний апендицит Pulmonary appendicitis

Хронічний апендицит Chronic appendicitis

Водянка апендикса Appendix dropsy

131 / 200
У лікарню наприкінці робочого дня доставлений робітник «гарячого» цеху, який скаржиться на головний біль, запаморочення, нудоту, загальну слабкість. Об’єктивно: свідомість збережена, шкірні покриви гіперемовані, сухі, гарячі на дотик. Частота серцевих скорочень - 130/хв. Дихання часте, поверхневе. Яке порушення процесів терморегуляції, найбільш імовірно, виникло у людини в даній ситуації"? At the end of the working day, a worker of a 'hot' workshop was brought to the hospital, who complains of headache, dizziness, nausea, general weakness. Objectively: consciousness is preserved, the skin is hyperemic, dry, hot to the touch. Heart rate - 130/min. Breathing is frequent, shallow. What violation of thermoregulation processes most likely occurred in a person in this situation?

Посилення теплопродукції без змін тепловіддачі Increased heat production without changes in heat output

Посилення тепловіддачі і зниження теплопродукції Increasing heat transfer and decreasing heat production

Посилення тепловіддачі і теплопродукції Increasing heat transfer and heat production

Зниження тепловіддачі Reduction of heat transfer

Зниження теплопродукції без змін тепловіддачі Reduction of heat production without changes in heat output

132 / 200
При розтині трупа чоловіка 48-ми років виявлено, що кістковий мозок пласких кісток, діафізів та епіфізів трубчастих кісток соковитий, сіро-червоний або сіро-жовтий гноєподібний (піоїдний кістковий мозок). Селезінка масою - 7 кг. На розрізі вона темно-червоного кольору, з ішемічними інфарктами. Всі лімфатичні вузли збільшені, м’які, сіро-червоного кольору. В печінці жирова дистрофія і лейкемічні інфільтрати. Який найбільш імовірний діагноз? During the autopsy of the corpse of a 48-year-old man, it was found that the bone marrow of flat bones, diaphyses and epiphyses of tubular bones, the juice is wrapped, gray-red or gray-yellow purulent (pyoid bone marrow). The spleen weighs 7 kg. On cross-section, it is dark red in color, with ischemic heart attacks. All lymph nodes are enlarged, soft, gray-red in color. Fatty dystrophy and leukemic infiltrates in the liver. What is the most likely diagnosis?

Хронічний мієлоїдний лейкоз Chronic myeloid leukemia

Мієломна хвороба Myeloma

Гострий лімфоїдний лейкоз Acute lymphoid leukemia

Гострий мієлоїдний лейкоз Acute myeloid leukemia

Лімфогранулематоз Lymphogranulomatosis

133 / 200
У 3-річної дитини тривале підвищення температури, збільшені лімфовузли, у крові - значне підвищення кількості лімфоцитів. Методом ІФА виявлено антиген віруса Епштейна-Бара. Який діагноз можна поставити на основі вказаного? A 3-year-old child has a prolonged fever, enlarged lymph nodes, blood - a significant increase in the number of lymphocytes. Epstein-Barr virus antigen was detected by ELISA method. What diagnosis can be made based on the above?

Герпетична аденопатія Herpetic adenopathy

Цитомегаловірусна інфекція Cytomegalovirus infection

Інфекційний мононуклеоз Infectious mononucleosis

Генералізована інфекція, викликана herpes-zoster Generalized infection caused by herpes-zoster

Лімфома Беркета Burkett's lymphoma

134 / 200
В бактеріологічну лабораторію поступив досліджуваний матеріал (промивні води, в’ялена риба домашнього приготування), взятий у хворого з підозрою на ботулізм. На яке поживне середовище слід зробити первинний посів матеріалу? The bacteriological laboratory received the researched material (wash water, dried home-cooked fish), taken from a patient with suspected botulism. On which nutrient medium should the primary sowing of the material be made?

Цукровий м’ясо-пептонний бульйон Sugar meat-peptone broth

Сироватковий агар Serum Agar

Цукровий м’ясо-пептонний агар Sugar meat-peptone agar

Цукрово-кров’яний агар Sugar-blood agar

Середовище Кітта-Тароцці Kitta-Tarozzi Environment

135 / 200
У юнака 18-ти років діагностовано хворобу Марфана. При дослідженні встановлено: порушення розвитку сполучної тканини, будови кришталика ока, аномалії серцево-судинної системи, арахнодактилія. Яке генетичне явище зумовило розвиток цієї хвороби? An 18-year-old man was diagnosed with Marfan's disease. The examination revealed: disorders of the development of connective tissue, the structure of the lens of the eye, anomalies of the cardiovascular system, arachnodactyly. What genetic phenomenon caused the development of this disease?

Плейотропія Pleiotropy

Комплементарність Complementarity

Множинний алелiзм Multiple allelism

Неповне домiнування Incomplete dominance

Кодомінування Codominance

136 / 200
У бактерій встановлений процес кон’югації, при якому між бактерiями утворюється цитоплазматичний місток, по якому з клітини-донора до клітини-реціпієнта надходять плазміди і фрагменти молекули ДНК. Яке значення цього процесу? Bacterials have a conjugation process in which a a cytoplasmic bridge through which plasmids and DNA molecule fragments arrive from the donor cell to the recipient cell. What is the significance of this process?

Сприяє активізації мутаційного процесу Contributes to the activation of the mutation process

Ліквідує небажані мутації Eliminates unwanted mutations

Підвищує гетерозиготність Increases heterozygosity

Забезпечує обмін речовинами між клітинами Ensures metabolism between cells

Забезпечує обмін генетичного матеріалу Ensures the exchange of genetic material

137 / 200
Дослідженнями останніх десятиліть встановлено, що безпосередніми «виконавцями» апоптозу в клітині є особливі ферменти - каспази. В утворенні одного з них бере участь цитохром С. Вкажіть його функцію в нормальній клітині: Research over the past decades has established that direct 'executors' apoptosis in the cell there are special enzymes - caspases. Cytochrome C is involved in the formation of one of them. State its function in a normal cell:

Компонент H + - АТФ-азной системи H + component of the ATPase system

Фермент дихального ланцюга переносу електронів Enzyme of the respiratory chain of electron transfer

Фермент ЦТК CTA Enzyme

Фермент бета-окислювання жирних кислот Enzyme beta-oxidation of fatty acids

Компонент піруватдегідрогеназної системи Component of pyruvate dehydrogenase system

138 / 200
У відділення реанімації був доставлений хворий з інфарктом міокарда. Який препарат необхідно ввести хворому для лікування больового шоку? A patient with a myocardial infarction was brought to the intensive care unit. What drug must be administered to the patient for the treatment of pain shock?

Налоксон Naloxone

Целекоксиб Celecoxib

Промедол Promedol

Анальгін Analgin

Парацетамол Paracetamol

139 / 200
Жінка 52-х років, хвора на рак молочної залози, пройшла курс променевої терапії. Розмір пухлини зменшився. Який з наведених механізмів ушкодження клітини найбільше обумовлює ефективність променевої терапії? A 52-year-old woman with breast cancer underwent radiation therapy. Size tumors decreased. Which of the following mechanisms of cell damage most determines the effectiveness of radiation therapy?

Тромбоз судин Sudin Thrombosis

Мутагенез Mutagenesis

Утворення вільних радикалів Formation of free radicals

Гіпертермія Hyperthermia

Лізис NK-клітинами Lysis by NK cells

140 / 200
Після операції на кишечнику у хворого з’явились симптоми отруєння аміаком за типом печінкової коми. Який механізм дії аміаку на енергозабезпечення ЦНС? After bowel surgery, the patient developed symptoms of ammonia poisoning according to the type hepatic coma. What is the mechanism of action of ammonia on the energy supply of the central nervous system?

Гальмування ЦТК в результаті зв’язування альфа-кетоглутарату CTC inhibition as a result of alpha-ketoglutarate binding

Гальмування гліколізу Inhibition of glycolysis

Інактивація ферментів дихального ланцюга Inactivation of respiratory chain enzymes

Гальмування бета-окиснення жирних кислот Inhibition of beta-oxidation of fatty acids

Роз’єднування окисного фосфорилювання Uncoupling oxidative phosphorylation

141 / 200
У пацієнта перфоративна виразка передньої стінки шлунка. В яке похідне очеревини попаде вміст шлунка? The patient has a perforated ulcer of the anterior wall of the stomach. What is the origin of the peritoneum will the contents of the stomach get in?

Чепцева сумка Cap Bag

Правий брижовий синус Right mesenteric sinus

Лівий брижовий синус Left mesenteric sinus

Печінкова сумка Liver bag

Передшлункова сумка Bag

142 / 200
При огляді хворої лікар-гінеколог відмітив симптоми запалення статевих шляхів, у мазку взятому із піхви, виявлено грушоподібні найпростіші з шипом, з передньої частини відходять джгутики, наявна ундулююча мембрана. Яке захворювання підозрює лікар у хворої? When examining the patient, the gynecologist noticed symptoms of inflammation of the genital tract, in the smear taken from the vagina, pear-shaped protozoa with a thorn were found, flagella depart from the front part, an undulating membrane is present. What disease does the doctor suspect in the patient?

Токсоплазмоз Toxoplasmosis

Кишковий трихомоноз Intestinal trichomoniasis

Лямбліоз Giardiasis

Урогенітальний трихомоноз Urogenital trichomoniasis

Балантидіоз Balantidiosis

143 / 200
Жінка звернулася до лікаря зі скаргами на утруднення рухів язика. Обстеження головного мозку за допомогою ЯМР показало, що у хворої крововилив в нижньому відділі довгастого мозку. Про пошкодження якого ядра довгастого мозку у хворої можна думати? The woman went to the doctor with complaints of difficulty in moving the tongue. Examination MRI of the brain showed that the patient had a hemorrhage in the lower part of the medulla oblongata. Which nucleus of the medulla oblongata can be thought to be damaged in the patient?

Ядро додаткового нерва Nucleus of additional nerve

Подвійне ядро Dual Core

Ядро під’язикового нерва Nucleus of the hypoglossal nerve

Одиноке ядро Lone Core

Нижнє слиновидільне ядро Inferior salivary nucleus

144 / 200
При мікроскопічному дослідженні виявляється паренхіматозний орган, в якому епітеліальні тяжі формують клубочкову, пучкову та сітчасту зони. Центральна частина органу представлена скупченнями хроматофінних клітин. Визначте орган: Microscopic examination reveals a parenchymal organ in which epithelial strands form the glomerular, bundle and reticular zones. The central part of the organ is represented by clusters of chromatin cells. Define the body:

Щитоподібна залоза Thyroid

Надниркова залоза Adrenal gland

Епіфіз Pineal gland

Печінка Liver

Гіпофіз Pituitary

145 / 200
У відділення реанімації поступив хворий після ДТП з однобічним пневмотораксом. Який вид дихання спостерігається у даному випадку? A patient was admitted to the intensive care unit after a road accident with unilateral pneumothorax. Which type of breathing is observed in this case?

Поверхневе рідке Surface liquid

Глибоке часте Deep Frequent

Поверхневе Superficial

Поверхневе часте Surface frequent

Асфіктичне Asphyxic

146 / 200
У хворої виявлена ракова пухлина шийки матки. У які регіонарні лімфатичні вузли можливе розповсюдження метастазів? The patient was diagnosed with a cancerous tumor of the cervix. Which regional lymph nodes possible spread of metastases?

Навколоміхурні і поперекові Circular and lumbar

Верхні і нижні брижові Upper and lower mesentery

Поперекові і ниркові Lumbar and renal

- -

Пахвинні і здухвинні Inguinal and iliac

147 / 200
При огляді порожнини рота дитини педіатр виявила наявність нижніх медіальних різців. Дитина розвивається нормально. Визначте її вік: During the examination of the child's oral cavity, the pediatrician found the presence of lower medial incisors. The child is developing normally. Determine her age:

8-9 місяців 8-9 months

10-12 місяців 10-12 months

6-7 місяців 6-7 months

13-14 місяців 13-14 months

- -

148 / 200
Серед антиатеросклеротичних препаратів, що застосовуються з метою профілактики та лікування атеросклерозу, є левостатин. Він діє шляхом: Among the antiatherosclerotic drugs used for prevention and treatment of atherosclerosis, there is levostatin. It works by:

Активації метаболізму холестерину Activations of cholesterol metabolism

Гальмування біосинтезу холестерину Cholesterol biosynthesis inhibition

Пригнічення всмоктування холестерину в кишківнику Inhibition of cholesterol absorption in the intestine

Стимулювання екскреції холестерину з організму Stimulation of cholesterol excretion from the body

Усіма наведеними шляхами All given paths

149 / 200
При зниженні активності ферментів антиоксидантного захисту посилюються процеси перекисного окиснення ліпідів клітинних мембран. При нестачі якого мікроелементу знижується активність глутатіонпероксидази? When the activity of antioxidant protection enzymes decreases, the processes increase peroxidation of cell membrane lipids. With a lack of which trace element, the activity of glutathione peroxidase decreases?

Мідь Copper

Селен Селен

Марганець Manganese

Молібден Molybdenum

Кобальт Cobalt

150 / 200
Хворий 75-ти років був оперований з приводу рака передміхурової залози, помер раптово на 4-ту добу після оперативного втручання. При розтині тіла померлого у просвітах головного стовбура і біфуркації легеневої артерії були виявлені і легко видалені крихкі маси темно-червоного кольору з тьмяною поверхнею. Такі ж маси знаходились у порожнині правого шлуночка серця. Який різновид порушення кровообігу призвів до раптової смерті хворого? A 75-year-old patient was operated on for prostate cancer, died suddenly on the 4th day after surgery. During the autopsy of the dead body, in the lumen of the main trunk and the bifurcation of the pulmonary artery, fragile masses of dark red color with a dull surface were found and easily removed. The same masses were located in the cavity of the right ventricle of the heart. What type of blood circulation disorder led to the patient's sudden death?

Тканинна емболія Tissue embolism

Парадоксальна емболія Paradoxical embolism

Тромбоз легеневої артерії Thrombosis of the pulmonary artery

Тромбоемболія легеневої артерії Thromboembolism of the pulmonary artery

!нфаркт міокарда !myocardial infarction

151 / 200
При обстеженні жінки 56-ти років, що хвора на цукровий діабет 1-го типу, виявлене порушення білкового обміну, що при лабораторному дослідженні крові проявляється аміноацидемією а клінічно - уповільненням загоєння ран і зменшенням синтезу антитіл. Який з перерахованих механізмів викликає розвиток аміноацидемії? During the examination of a 56-year-old woman with type 1 diabetes, it was found a violation of protein metabolism, which is manifested by aminoacidemia during a laboratory blood test, and clinically by a slowdown in wound healing and a decrease in the synthesis of antibodies. Which of the listed mechanisms causes the development of aminoacidemia?

Гіперпротеїнемія Hyperproteinemia

Зменшення концентрації амінокислот у крові Decreased concentration of amino acids in blood

Підвищення протеолізу Proteolysis increase

Збільшення ліпопротеїдів високої щільності High-density lipoprotein increase

Підвищення онкотичного тиску в плазмі крові Increased oncotic pressure in blood plasma

152 / 200
При недостатності вітаміну C спостерігається порушення структури колагенових волокон. Яка стадія їх синтезу порушується при цьому? With vitamin C deficiency, there is a violation of the structure of collagen fibers. What stage of their synthesis is disturbed at the same time?

Відщеплення сигнальних олігопептидів Separation of signal oligopeptides

Утворення гідроксипроліну і гідроксилізину Formation of hydroxyproline and hydroxylysine

Приєднання глюкозних і галактозних залишків Addition of glucose and galactose residues

Утворення гама-карбоксиглутамату Formation of gamma-carboxyglutamate

Агрегація тропоколагену і утворення фібрил Tropocollagen aggregation and fibril formation

153 / 200
У людини в результаті патологічного процесу збільшена товщина гематоальвеолярного бар’єру. Безпосереднім наслідком цього буде зменшення: In a person, as a result of a pathological process, the thickness of the hematoalveolar the barrier A direct consequence of this will be a reduction:

Кисневої ємності крові Blood oxygen capacity

Альвеолярної вентиляції легень Alveolar lung ventilation

Дифузійної здатності легень Diffusing capacity of lungs

Хвилинного об’єму дихання Minute respiratory volume

Резервного об’єму видиху Exhalation reserve volume

154 / 200
Недостатність в організмі мікроелементу селену проявляється кардіоміопатією. Імовірною причиною такого стану є зниження активності такого селенвмісного ферменту: Deficiency of the trace element selenium in the body is manifested by cardiomyopathy. The likely cause of this condition is a decrease in the activity of this selenium-containing enzyme:

Глутатіонпероксидаза Glutathione peroxidase

Каталаза Catalase

Цитохромоксидаза Cytochrome oxidase

Лактатдегідрогеназа Lactate dehydrogenase

Сукцинатдегідрогеназа Succinate dehydrogenase

155 / 200
У хворого після перенесеного ішемічного інсульту стали неможливими довільні рухи у правих кінцівках, спостерігається гіперрефлексія. Під час пальпації визначається підвищений тонус м’язів кінцівок. Яка форма порушення рухової функції має місце у хворого? After an ischemic stroke, the patient became unable to make voluntary movements in right limbs, hyperreflexia is observed. During palpation, an increased tone of the muscles of the limbs is determined. What form of impaired motor function does the patient have?

Периферичний параліч Peripheral paralysis

Центральний параліч Central paralysis

Тетанія Tetany

Периферичний парез Peripheral paresis

Мозочкова атаксія Cerebellar ataxia

156 / 200
У жінки, що тривало приймала антибіотики з приводу кишкової інфекції, розвинулось ускладнення з боку слизової порожнини рота у вигляді запального процесу і білого нальоту, у якому під час бактеріологічного дослідження були виявлені дріжджеподібні грибки Candida albicans. Який з перерахованих препаратів показаний для лікування цього ускладнення? A woman who took antibiotics for a long time for an intestinal infection developed complications from the mucous membrane of the mouth in the form of an inflammatory process and a white plaque, in which yeast-like fungi Candida albicans were detected during a bacteriological examination. Which of the listed drugs is indicated for the treatment of this complication?

Поліміксин Polymixin

Бісептол Biseptol

Фуразолідон Furazolidone

Тетрациклін Tetracycline

Флуконазол Fluconazole

157 / 200
Досліджуються клітини червоного кісткового мозку людини, які належать до клітинного комплексу, що постійно діляться. Який процес забезпечує генетичну ідентичність цих клітин: Human red bone marrow cells, which belong to the cellular complex that is constantly dividing. What process ensures the genetic identity of these cells:

Мейоз Meiosis

Трансплантація Transplant

Мітоз Мітоз

Мутація Mutation

Репарація Reparation

158 / 200
Хвора 57-ми років для лікування гіпертонічної хвороби тривалий час приймала анаприлін. Побічні ефекти спонукали пацієнтку відмовитись від прийому препарату, що призвело до розвитку гіпертонічного кризу і нападу стенокардії. Як називається ускладнення, яке виникло? A 57-year-old patient has been taking anaprilin for a long time to treat hypertension. Side effects prompted the patient to stop taking the drug, which led to the development of a hypertensive crisis and an attack of angina pectoris. What is the name of the complication that occurred?

Звикання Addiction

Лікарська залежність Drug addiction

Сенсибілізація Sensitization

Синдром відміни Withdrawal Syndrome

Тахіфілаксія Tachyphylaxis

159 / 200
При лабораторному дослідженні крові пацієнта виявлено, що вміст білків у плазмі становить 40 г/л. Як це впливає на транскапілярний обмін води в мікроциркуляторному руслі? During the laboratory examination of the patient's blood, it was found that the protein content in the plasma is 40 g/l. How does it affect the transcapillary exchange of water in the microcirculatory channel?

Обмін не змінюється Exchange does not change

Збільшуються фільтрація і реабсорбція Filtration and reabsorption are increasing

Зменшується фільтрація, збільшується реабсорбція Filtration decreases, reabsorption increases

Збільшується фільтрація, зменшується реабсорбція Filtration increases, reabsorption decreases

Зменшуються фільтрація і реабсорбція Filtration and reabsorption are decreasing

160 / 200
У хворого спостерігається тремтіння рук, що пов’язане з хворобою Паркінсона. Дефіцит якого медіатора в стріопалідарних структурах призводить до таких симптомів? The patient has hand tremors associated with Parkinson's disease. Deficiency which mediator in striopallidary structures leads to such symptoms?

ГАМК ГАМК

Субстанція Р Substance P

Серотонін Serotonin

Норадреналін Noradrenaline

Дофамін Dopamine

161 / 200
У хворого на жовчно-кам’яну хворобу спостерігаються ознаки холемічного синдрому. Який симптом з перерахованих зумовлений відсутністю надходження жовчі у кишечник? A patient with gallstones has signs of cholemic syndrome. Which of the following symptoms is caused by the absence of bile in the intestines?

Гіпотонія Hypotonia

Шкірний свербіж Itchy skin

Стеаторея Steatorrhea

Астенія Asthenia

Брадикардія Bradycardia

162 / 200
У людини після гострої крововтрати виникло відчуття спраги. Зміна якого гомеостатичного параметру викликала це відчуття? A person felt thirsty after acute blood loss. The change of which homeostatic parameter caused this feeling?

Підвищення осмотичного тиску рідин організму Increased osmotic pressure of body fluids

Зниження онкотичного тиску рідин організму Reduction of oncotic pressure of body fluids

Зниження осмотичного тиску рідин організму Decreased osmotic pressure of body fluids

Зменшення об’єму позаклітинної рідини Decreasing the volume of extracellular fluid

Підвищення онкотичного тиску рідин організму Increased oncotic pressure of body fluids

163 / 200
Студент використав консервовану донорську кров для визначення часу її зсідання. Однак, будь-якого позитивного результату він отримати не зміг. Причиною цього є відсутність в крові: The student used canned donor blood to determine its clotting time. However, he could not get any positive result. The reason for this is the absence in the blood:

Іонізованого кальцію Ionized calcium

Тромбопластину Thromboplate

Фактора Хагемана Hageman Factor

Фі6риногєну Fi6rinogenu

Вітаміну K Vitamin K

164 / 200
Введення тварині екстракту тканини передсердя посилює виділення натрію з сечею. Дія якої біологічно активної речовини стала причиною такого стану? The administration of an extract of atrial tissue to an animal increases the excretion of sodium in the urine. Action What biologically active substance caused this condition?

Серотонін Serotonin

Адреналін Adrenaline

Натрійуретичний гормон Natriuretic hormone

Ілюкокортикоїд Ilucocorticoid

Калійкреїн Potassiumcrein

165 / 200
Хворій жінці із захворюванням нирок, що супроводжується вираженими набряками, призначили діуретичний препарат, що пригнічує реабсорбцію в нирках іонів Na+ і води, посилює виведення нирками іонів K+ і Mg++, викликає гіперурикемію, зумовлює потужний діуретичний ефект. Назвіть цей препарат: A sick woman with kidney disease, accompanied by severe swelling, prescribed a diuretic drug that suppresses the reabsorption of Na+ ions and water in the kidneys, increases the excretion of K+ and Mg++ ions by the kidneys, causes hyperuricemia, and causes a powerful diuretic effect. Name this drug:

Спіронолактон Spironolactone

Аллопуринол Allopurinol

Діакарб Diakarb

Тріамтерен Triamterene

Фуросемід Furosemide

166 / 200
Хворому призначили антибіотик хлорамфенікол (левомецитин), який порушує у мікроорганізмів синтез білку шляхом гальмування процесу: The patient was prescribed the antibiotic chloramphenicol (levomecitin), which violates microorganisms protein synthesis by inhibiting the process:

Процесінг Processing

Транскрипція Transcription

Ампліфікація генів Gene amplification

Утворення полірибосом Formation of polyribosomes

Елонгація трансляції Broadcast elongation

167 / 200
До офтальмолога звернулася жінка зі скаргами на шкірний свербіж і набряклість повік. При обстеженні виявлено членистоноге червоподібної форми величиною 0,4 мм. Суцільний щиток вкриває передню частину тіла, тіло має поперечну смугастість. Ноги короткі, лапки з двома кігтиками. Який діагноз може встановити лікар? A woman turned to an ophthalmologist with complaints of skin itching and swelling of the eyelids. During the examination, a worm-shaped arthropod with a size of 0.4 mm was found. A solid shield covers the front part of the body, the body has transverse stripes. Legs are short, paws with two claws. What diagnosis can the doctor make?

Короста Scabies

Фтиріоз Phthyriosis

Факультативний міаз Facultative myiasis

Педікульоз Pediculosis

Демодекоз Demodecosis

168 / 200
У хворого 69-ти років на шкірі в ділянці нижньої повіки з’явилося невелике бляшкоподібне утворення з наступним виразкуванням, яке було оперативно видалене. При мікроскопічному дослідженні утворення: в дермі шкіри комплекси з атипових епітеліальних клітин; периферії клітини розташовані перпендикулярно до базальної мембрани. Клітини темні, призматичної полігональної форми з гіперхромними ядрами з частими мітозами. Іноді зустрічаються утворення, подібні до волосяного фолікула. Яка гістологічна форма раку у хворого? A 69-year-old patient had a small rash on the skin of the lower eyelid a plaque-like formation followed by ulceration, which was surgically removed. Under microscopic examination of the formation: complexes of atypical epithelial cells in the dermis of the skin; the periphery of the cell is located perpendicular to the basement membrane. Cells are dark, prismatic polygonal in shape with hyperchromic nuclei with frequent mitoses. Sometimes there are formations similar to a hair follicle. What is the histological form of cancer in the patient?

Плоскоклітинний рак з ороговінням Squamous cell carcinoma with keratinization

Плоскоклітинний рак без ороговіння Squamous cell carcinoma without keratinization

Базально-клітинний рак Basal cell carcinoma

Аденокарцинома Adenocarcinoma

Недиференційований рак Undifferentiated cancer

169 / 200
Після перелому нижньої щелепи постраждалий відзначає втрату чутливості шкіри у ділянці підборіддя і нижньої губи. Який нерв був пошкоджений? After a fracture of the lower jaw, the victim notes a loss of skin sensitivity in area of the chin and lower lip. What nerve was damaged?

Maxillaris Maxillaris

Infraorbitalis Infraorbital

Buccalis Buccalis

Mentalis Mentalis

Facialis Facial

170 / 200
У молодого чоловіка внаслідок подразнення сонячного сплетення запальним процесом (солярит) підвищена функціональна активність залоз шлунка, що виражається, зокрема, у збільшенні продукції хлоридної кислоти. Яка з вказаних нижче речовин викликає гіперхлоргідрію у даному випадку? A young man due to irritation of the solar plexus by an inflammatory process (solaritis) increased functional activity of the stomach glands, which is expressed, in particular, in an increase in the production of hydrochloric acid. Which of the following substances causes hyperchlorhydria in this case?

Гастрин Гастрин

Гастроінгібуючий пептид Gastroinhibitory peptide

Глюкагон Glucagon

Урогастрон Urogastron

Калікреїн Kallikrein

171 / 200
У альпініста, що піднявся на висоту 5200 м, розвинувся газовий алкалоз. Що є причиною його розвитку? A mountaineer who climbed to a height of 5200 m developed gas alkalosis. What is the cause its development?

Гіпервентиляція легенів Hyperventilation of the lungs

Гіповентиляція легенів Hypoventilation of the lungs

Гіпоксемія Hypoxemia

Гіпероксемія Hyperoxemia

Зниження температури навколишнього середовища Decreasing ambient temperature

172 / 200
Реакції міжмолекулярного транспорту одновуглецевих радикалів є необхідними для синтезу білків та нуклеїнових кислот. З якого з наведених нижче вітамінів утворюється кофермент, потрібний для вказаних вище реакцій? Reactions of intermolecular transport of one-carbon radicals are necessary for synthesis of proteins and nucleic acids. From which of the following vitamins is the coenzyme required for the above reactions formed?

Тіамін Thiamine

Аскорбінова кислота Ascorbic acid

Пантотенова кислота Pantothenic Acid

Фолієва кислота Folic acid

Рибофлавін Riboflavin

173 / 200
Прокаріотичні та еукаріотичні клітини характеризуються здатністю до по- ділу. Поділ прокаріотичних клітин відрізняється від поділу еукаріотичних, але існує молекулярний процес, який лежить в основі цих поділів. Який це процес? Prokaryotic and eukaryotic cells are characterized by the ability to divide. prokaryotic cell division differs from eukaryotic cell division, but there is a molecular process that underlies these divisions. What is the process?

Репарація Reparation

Транскрипція Transcription

Ампліфікація генів Gene amplification

Реплікація ДНК DNA Replication

Трансляція Broadcast

174 / 200
Для проведення анальгезії наркотичний анальгетик застосували з препаратом бензодіазепінового ряду. Який засіб використали для потенціювання анальгезії? For analgesia, a narcotic analgesic was used with the drug benzodiazepine series. What tool was used to potentiate analgesia?

Карбамазепін Carbamazepine

Імізин Imisin

Хлорпротіксен Chlorprothixene

Трифтазин Triftazine

Діазепам Diazepam

175 / 200
Молода людина 25-ти років споживає надмірну кількість вуглеводів (600 г на добу), що перевищує її енергетичні потреби. Який процес буде активуватися в організмі людини уданому випадку? A 25-year-old young man consumes an excessive amount of carbohydrates (600 g per day), which exceeds its energy needs. What process will be activated in the human body in a successful case?

Окиснення жирних кислот Oxidation of fatty acids

Ліполіз Lipolysis

Гліколіз Glycolysis

Глюконеогенез Gluconeogenesis

Ліпогенез Lipogenesis

176 / 200
Юнак 15-ти років скаржиться на загальну слабкість, запаморочення, швидку стомлюваність. При обстеженні виявлено еритроцити зміненої форми, кількість їх знижена. Попередній діагноз: серпоподібно-клітинна анемія. Який тип мутації зумовлює розвиток цього патологічного стану? A 15-year-old boy complains of general weakness, dizziness, rapid fatigue During the examination, erythrocytes of a changed shape were found, their number was reduced. Previous diagnosis: sickle cell anemia. What type of mutation causes the development of this pathological condition?

Делеція Deletion

Мутація зсуву рамки зчитування Reading frame shift mutation

Інверсія Inversion

Точкова мутація Point Mutation

Хромосомна аберація Chromosomal aberration

177 / 200
Хворому із больовим синдромом в суглобах постійно призначають аспірин. Який з перерахованих ферментів він пригнічує? A patient with joint pain is constantly prescribed aspirin. Which of the does it suppress the listed enzymes?

Фосфоліпаза А2 Phospholipase A2

Ліпооксигеназа Lipoxygenase

Циклооксигеназа Cyclooxygenase

Фосфоліпаза Д Phospholipase D

Фосфоліпаза С Phospholipase C

178 / 200
Препарати групи вітаміну В2 призначають при захворюваннях шкіри. Завдяки наявності якої структури в його складі визначається здатність до окиснення-відновлення? Vitamin B2 drugs are prescribed for skin diseases. Thanks to the presence of which structure in its composition determines the ability to oxidation-reduction?

Аденін Аденін

Ізоалоксазин Isoalloxazine

Залишок фосфорної кислоти Phosphoric acid residue

Рибозофосфат Ribose Phosphate

Рибітол Ribitol

179 / 200
Під час автомобільної аварії людина отримала сильний удар в епігастральну ділянку, внаслідок чого виникла зупинка серця. Що могло стати причиною таких змін серцевої діяльності? During a car accident, a person received a strong blow to the epigastric area, resulting in cardiac arrest. What could have caused such changes in heart activity?

Збільшене виділення кортизолу Increased secretion of cortisol

Збільшене виділення адреналіну Increased release of adrenaline

Підвищення тонусу блукаючого нерва Increasing the tone of the vagus nerve

Підвищення тонусу симпатичної нервової системи Increasing the tone of the sympathetic nervous system

Збільшене виділення альдостерону Increased release of aldosterone

180 / 200
У хворого з синдромом Іценко-Кушинга спостерігаються стійка гіперглікемія та глюкозурія. Синтез та секреція якого гормону збільшені у цього хворого? A patient with Itsenko-Cushing syndrome has persistent hyperglycemia and glucosuria Synthesis and secretion of which hormone is increased in this patient?

Адреналін Adrenaline

Альдостерон Aldosterone

Глюкагон Glucagon

Тироксин Thyroxine

Кортизол Кортизол

181 / 200
У хворого має місце хронічний запальний процес мигдаликів. За рахунок якого біохімічного процесу у вогнищі запалення підтримується концентрація НАДФН, необхідного для реалізації механізму фагоцитозу? The patient has a chronic inflammatory process of the tonsils. Due to which biochemical process in the focus of inflammation, the concentration of NADPH, necessary for the implementation of the mechanism of phagocytosis, is maintained?

Пентозо-фосфатний шлях Pentose phosphate pathway

Цикл Кребса Krebs Cycle

Цикл Корі Cory Cycle

Орнітиновий цикл Ornithine cycle

Гліколіз Glycolysis

182 / 200
Щоденно в організмі людини 0,5% всього гемоглобіну перетворюється на метгемоглобін. Який фермент, що міститься в еритроцитах, каталізує відновлення метгемоглобіну до гемоглобіну? Every day in the human body, 0.5% of all hemoglobin is converted into methemoglobin. What enzyme, contained in erythrocytes, catalyzes the reduction of methemoglobin to hemoglobin?

Метгемоглобінредуктаза Methemoglobin reductase

Білівердинредуктаза Biliverdin reductase

Метгемоглобінтрансфераза Methemoglobintransferase

Глюкуронілтрансфераза Glucuronyltransferase

Гемоксигеназа Hemoxygenase

183 / 200
У хворого з гемолітичною анемією виявлено дефіцит піруваткінази в еритроцитах. За цих умов причиною розвитку гемолізу еритроцитів є: A patient with hemolytic anemia was diagnosed with pyruvate kinase deficiency in erythrocytes. conditions, the cause of hemolysis of erythrocytes is:

Зменшення активності Na+, К+ -АТФ-ази Decreased activity of Na+, K+ -ATP-ase

Генетичні дефекти глікофорину А Glycophorin A genetic defects

Надлишок К + в еритроцитах Excess K + in erythrocytes

Нестача Na+ в еритроцитах Lack of Na+ in erythrocytes

Дефіцит спектрину Spectrin deficiency

184 / 200
Хвора 37-ми років померла під час нападу експіраторної задухи, що був спричинений контактом з екзогенним алергеном (пилок амброзії). При гістологічному дослідженні в просвіті бронхів спостерігаються скупчення слизу, в стінці бронхів багато тучних клітин (лаброцитів), більшість з яких у стані дегрануляції, багато еозинофілів. До патогенезу якого типу реакцій гіперчутливості можна віднести описані зміни? A 37-year-old patient died during an attack of expiratory asphyxia caused by contact with an exogenous allergen (ragweed pollen). During histological examination, accumulation of mucus is observed in the lumen of the bronchi, in the wall of the bronchi there are many mast cells (labrocytes), most of which are in a state of degranulation, and there are many eosinophils. The described changes can be attributed to the pathogenesis of which type of hypersensitivity reactions?

IV типу (клітинна цитотоксичність) Type IV (cellular cytotoxicity)

I типу (анафілактична) Type I (anaphylactic)

II типу (антитілозалежна) Type II (antibody dependent)

III типу (імунокомплексна) Type III (immunocomplex)

V типу (гранулематоз) Type V (granulomatosis)

185 / 200
У хворого запалення присереднього надвиростка плечової кістки (епіконділіт). Який нерв залучений у процес? The patient has inflammation of the medial epicondyle of the humerus (epicondylitis). Which nerve involved in the process?

N. medianus N. medianus

N. musculocutaneus N. musculocutaneus

N. ulnaris N. ulnaris

N. axillaris N. axillaris

N. radialis N. radialis

186 / 200
Який вітамінний препарат доцільно призначити пацієнту зі скаргами на зниження гостроти зору в сутінках, сухість шкіри, часті застудні захворювання, ламкість волосся? Which vitamin preparation should be prescribed to a patient with complaints of decreased visual acuity at dusk, dry skin, frequent colds, brittle hair?

Пантотенова кислота Pantothenic Acid

Нікотинова кислота Nicotinic acid

Ретинолу ацетат Retinol Acetate

Тіаміну хлорид Thiamine chloride

Ергокальциферол Ergocalciferol

187 / 200
43-х річний капітан корабля скаржиться на прояви сезонної алергії. Який лікарський засіб, що не має снодійного ефекту, можна йому призначити? A 43-year-old ship captain complains of seasonal allergies. What medicine can he be prescribed a drug that does not have a hypnotic effect?

Супрастин Suprastin

Ванкоміцин Vancomycin

Дипразин Diprazine

Лоратадин Loratadine

Мікосептин Mycoseptin

188 / 200
У жінки 52-х років артеріальна гіпертензія ускладнилась правобічною геміплегією і втратою мови. Яка зона головного мозку є імовірно найбільш ураженою? In a 52-year-old woman, arterial hypertension was complicated by right-sided hemiplegia and loss of language Which area of the brain is likely to be most affected?

Ліва передня звивина Left anterior gyrus

Права передня звивина Right anterior gyrus

Потилична частка Occipital lobe

Ліва передня звивина і ліва скронева частка Left frontal gyrus and left temporal lobe

Ліва скронева частка Left temporal lobe

189 / 200
Хворому на хронічний бронхіт призначили муколітичний препарат, що підвищує синтез сурфактанту. Який препарат був використаний? A patient with chronic bronchitis was prescribed a mucolytic drug that increases the synthesis surfactant. What drug was used?

Мукалтин Mukaltin

Фенотерол Fenoterol

Амброксол Ambroxol

Лібексин Libexin

Кодеїн Codeine

190 / 200
Ефективна діагностика носійства збудників кишкових інфекцій ґрунтується на виявленні антитіл до певних антигенів бактерій в реакції непрямої гемаглютинації. Який стандартний препарат слід застосувати у цій реакції'? Effective diagnosis of carriers of pathogens of intestinal infections is based on the detection of antibodies to certain antigens of bacteria in the reaction of indirect hemagglutination. Which the standard drug should be used in this reaction'?

Монорецепторні діагностичні сироватки Monoreceptor diagnostic sera

Моноклональні антитіла Monoclonal antibodies

Еритроцити барана й гемолітичну сироватку Ram erythrocytes and hemolytic serum

Еритроцитарні діагностикуми з адсорбованими антигенами бактерій Erythrocyte diagnostics with adsorbed bacterial antigens

Антитіла проти імуноглобулінів основних класів Antibodies against immunoglobulins of the main classes

191 / 200
На розтині тіла померлого, який страждав на гіпертонічну хворобу, у лівій гемісфері мозку виявлена порожнина округлої форми 4х5 см із іржавою стінкою, заповнена жовтуватою прозорою рідиною. Назвіть патологію, яка розвинулась у головному мозку хворого: On the autopsy of the body of the deceased, who suffered from hypertensive disease, in the left hemisphere a 4x5 cm round cavity with a rusty wall filled with a yellowish transparent liquid was found in the brain. Name the pathology that developed in the patient's brain:

Геморагічне просякнення Hemorrhagic impregnation

Гематома Hematoma

Кіста Кіста

Немічний інфаркт Nemic heart attack

Абсцес Abscess

192 / 200
Клінічні ознаки хвороби у пацієнта почалися через 24 години після вживання рибних консервів і включали нудоту, слабкість, сухість у роті. Потім приєдналися двоїння зображення, афагія, проблеми з диханням. Чим обумовлена така симптоматика при ботулізмі? Clinical signs of the disease in the patient began 24 hours after eating fish canned food and included nausea, weakness, dry mouth. Then came double vision, aphagia, breathing problems. What causes such symptoms in botulism?

Вплив ендотоксину Effect of endotoxin

Розмноження збудника у ШКТ Multiplication of the pathogen in the gastrointestinal tract

Вплив нейротоксину Neurotoxin exposure

Активація цАМФ Activation of cAMP

Вплив ентеротоксину Enterotoxin exposure

193 / 200
У хворого з підозрою на «озену» з носоглотки були виділені грамнегативні палички, які утворювали капсулу на поживному середовищі. Які мікроорганізми спричинили хворобу? Gram-negative bacilli were isolated from the nasopharynx of a patient with suspicion of 'ozen' formed a capsule on a nutrient medium. What microorganisms caused the disease?

Сальмонели Salmonella

Мікоплазми Mycoplasma

Шигели Шигели

Клебсієли Klebsiels

Хламідії Chlamydia

194 / 200
До травматологічного пункту доставлено хворого з пошкодженням м’язів нижніх кінцівок. За рахунок яких клітин можлива репаративна регенерація м’язових волокон і відновлення функції м’язів? A patient with damage to the muscles of the lower limbs At the expense of which cells is possible reparative regeneration of muscle fibers and restoration of muscle function?

Плазмоцити Plasmocytes

Ендотеліоцити Endotheliocytes

Міосателітоцити Myosatellitocytes

Адипоцити Adipocytes

Фібробласти Fibroblasts

195 / 200
У чоловіка 25-ти років на 4-й день після крововтрати в крові зросла кількість поліхроматофільних еритроцитів, ретикулоцитів і навіть з’явились поодинокі нормобласти. Про що свідчить такі гематологічні зміни? In a 25-year-old man, on the 4th day after blood loss, the number of polychromatophilic erythrocytes, reticulocytes, and even single normoblasts appeared in the blood. What does this indicate? hematological changes?

Зменшено об’єм плазми крові Decreased blood plasma volume

Процеси регенерації випереджають дозрівання клітин Regeneration processes precede cell maturation

Ослаблені процеси еритропоезу Weakened processes of erythropoiesis

Знижена продукція еритропоезу в нирках Decreased production of erythropoiesis in the kidneys

Підвищена спорідність гемоглобіну до кисню Increased affinity of hemoglobin to oxygen

196 / 200
Хворому на туберкульоз легень призначений препарат першого ряду, який спричинив розвиток невриту лицьового нерва та порушення рівноваги. Вкажіть цей лікарський засіб: A patient with pulmonary tuberculosis was prescribed a first-line drug that caused development of neuritis of the facial nerve and imbalance. Specify this medicine:

Бісептол Biseptol

Ізоніазид Isoniazid

Левоміцетин Levomycetin

Цефазолін Cefazolin

Фуразолідон Furazolidone

197 / 200
Хірург під час операції на щитоподібній залозі перев’язав верхню щитоподібну артерію. Гілку якої судини перев’язав лікар? The surgeon ligated the superior thyroid artery during thyroid surgery. The branch of which blood vessel did the doctor tie off?

A. lingualis A. lingual

A. carotis externa A. carotid externa

A. facialis A. facialis

A. pharyngea ascendens A. pharyngeal ascending

A. carotis interna A. internal carotid artery

198 / 200
Батьки - глухонімі, але глухота у дружини залежить від аутосомно-рецесивного гена, а у чоловіка виникла внаслідок тривалого прийому антибіотиків у дитинстві. Яка імовірність народження глухої дитини в родині, якщо батько гомозиготний за аллелю нормального слуху? The parents are deaf and mute, but the wife's deafness depends on an autosomal recessive gene, and the man's disease arose as a result of long-term use of antibiotics in childhood. What is the probability of a deaf child being born in a family if the father is homozygous for the normal hearing allele?

100% 100%

75% 75%

10% 10%

0% 0%

25% 25%

199 / 200
До лікарні потрапив футболіст з ушкодженням поверхневого пахвинного кільця та розривом двох ніжок, що його обмежують. Похідним якої анатомічної структури вони є? A football player was admitted to the hospital with an injury to the superficial inguinal ring and the gap between the two legs that limit it. What anatomical structure are they derived from?

Міжножкові волокна Interpedicular fibers

Власна фасція живота Own abdominal fascia

Апоневроз зовнішнього косого м’яза живота Aponeurosis of the external oblique muscle of the abdomen

Апоневроз поперечного м’яза живота Aponeurosis of the Abdominal Cross

Апоневроз внутрішнього косого м’яза живота Aponeurosis of the internal oblique muscle of the abdomen

200 / 200
У потерпілого перелом у ділянці бічної поверхні променево-зап’ясткового суглоба. Де найімовірніше відбувся перелом? The victim has a fracture in the area of the lateral surface of the radiocarpal joint. Where did the fracture most likely occur?

Гачкувата кістка Hookbone

Головчаста кістка Scapula

Шилоподібний відросток ліктьової кістки Spinal process of the ulna

Шилоподібний відросток променевої кістки Spinal process of the radius

Нижня третина плечової кістки Lower third of humerus